PRACTICE QUESTIONS FOR MEDSURG FINAL

अब Quizwiz के साथ अपने होमवर्क और परीक्षाओं को एस करें!

which esophageal disorder is described as a precancerous lesion and is associated with GERD? A. Achalasia B. Barrett's esophagus C. esophageal strictures D. esophageal diverticula

B

A nurse is caring for a client who has benign prostatic hyperplasia (BPH). Which of the following is an expected finding? Urge incontinence. Critically elevated prostate-specific antigen (PSA) level. Difficulty starting the flow of urine. Painful urination.

3. Difficulty starting the flow of urine. Hesitancy, or difficulty starting the flow of urine, is an expected finding of BPH. Overflow rather than urge incontinence is an expected finding of BPH. A critically elevated PSA level is associated with prostate cancer rather than BPH. Painful urination is not an expected finding of BPH unless it is associated with a complication such as a urinary tract infection.

What initial physical assessment finding would the nurse expect to be present in a patient witha cute left-sided heart failure? A. bubbling crackles and tachycardia B. hepatosplenomegaly and tachypnea C. peripheral edema and cool, diaphoretic skin D. frothy, blood-tinged sputum, distended jugular veins

A

which patient is at the HIGHEST risk of having a gastric ulcer? A. 55 year old female smoker with nasaeu/vomiting B. 45 year old female admitted for illicit drug detoxification C. 27 year old male who is divorced and has back pain D. 37 year old male smoker who has fallen

A

When teaching the patient with type 1 diabetes, what should the nurse emphasize as a major advantage to using the insulin pump? A. Tight glycemic control B. errors in insulin dosing are less likely to occur C. Complications of insulin therapy are prevented D. Frequent blood glucose monitoring is unneccessary

A.

With what are the postoperative respiratory complications of atelectasis and aspiration of gastric contents associated? A. hypoxemia B. hypercapnia C. hypoventilation D. airway obstruction

A.

The nurse uses many precautions during IV administration of vesicant chemotherapy agents primarily to prevent A. septicemia B. extravasation C. catheter occlusion D. anaphylactic shock

B

A patient is scheduled for bilateral adrenalectomy. During the postoperative period, the nurse would expect administration of corticosteroids to be a. reduced to promote wound healing b. withheld until symptoms of hypocortisolism appear c. increased to promote an adequate response to the stress of surgery d. reduced because excessive hormones are released during surgical manipulation of the glands

C

An increase in which blood cell indicates an increased rate of erythropoiesis? A. basophil B. monocyte C. reticulocyte D. lymphocyte

C

Following a splenectomy for the treatment of ITP, what lab test result would the nurse expect to find? A. decr RBCs B. Decr WBCs C. increased platelets D. incr immunoglobulins

C

Glomerulonephritis is characterized by glomerular damage caused by A. growth of microorganism B. release of bacterial substances toxic to glomeruli C. accumulation of immune complexes D. hemolysis of red blood cells

C

The patient's lab results show a marked decrease in RBCs, WBCs, and platelets. What term would the nurse use when describing this to the healthcare provider? A. hemolysis B. leukopenia C. Pancytopenia D. thrombocytosis

C

what causes the pain that occurs during myocardial ischemia? A.death of myocardial tissue B. dysrhythmias caused by cellular irritabiltiy C. Lactic acid accumulation D. elevated pressure in the ventricles and pulmonary vessels

C

what primarily determines the neurologic functions that are affected by a stroke? A. amount of tissue area involved B. rapidity of symptoms C. the brain area perfused by the affected artery D. the presence or absence of collateral circulation

C

when teaching a patient with a seizure disorder about the medication regimen, what is it most important for the nurse to emphasize? A. the patient should increase the dose of medication if stress is increased B. most OTC and prescription drugs are safe to take with anti-seizure durgs C. stopping the medication abruptly may increase the intensity and frequency of seizures D. if gingival hypertrophy occurs, the HCP should be notified and the drug may be changed

C

A nurse is caring for a client who has peptic ulcer disease. The nurse knows to monitor the client for which of the following findings as an indication of the complication of gastrointestinal perforation? Hyperactive bowel sounds Sudden abdominal pain Increased blood pressure Bradycardia

2. Sudden abdominal pain Classic indications of gastrointestinal perforation include sudden sharp abdominal pain, with a rigid abdomen, declining peristalsis, and progression to septicemia and hypovolemic shock.

A nurse is discussing the findings of a urinary tract infection (UTI) with a newly licensed nurse. Which of the following should the nurse identify as a finding specifically associated with a UTI in the older adult client? Urinary retention. Low back pain. Incontinence. Confusion.

4. Confusion. Confusion is a clinical finding of UTIs specifically associated with the older adult client. All other options are common clinical findings in clients of all ages.

After a woman had a right breast mastectomy, her right arm became severely swollen. What hematologic problem caused this? A. lymphedema B. right sided heart failure C. wound on her right hand D. refusal to use her right arm

A

Which urine specific gravity would indicate that the patient is receiving excessive IV fluid? A. 1.002 B. 1.010 C. 1.025 D. 1.033

A

A nurse is assessing a client who has an acute myocardial infarction (MI). Which of the following clinical manifestations should the nurse expect to find? (Select all that apply.) Orthopnea Headache Nausea Tachycardia Diaphoresis

3, 4 & 5. Nausea, Tachycardia and Diaphoresis Orthopnea is a manifestation of heart failure, which can develop from a myocardial infarction, but it is not a common manifestation of acute MI. Chest pain and sometimes jaw and shoulder pain, not headache, are classic manifestations of acute MI. Nausea, vomiting, tachycardia, dysrhythmias profuse sweating and anxiety are also classic manifestations of acute MI.

A nurse is caring for a client who is 1 day postoperative following a left radical mastectomy for breast cancer. Which of the following behaviors should alert the nurse to the possibility that the client is having difficulty adjusting to the loss of her breast? Refusing to look at the dressing or surgical incision Asking for pain medication every 3 hr Asking questions about the information on her postoperative care pamphlet Performing arm exercises once or twice a day

Refusing to look at the dressing or surgical incision Clients who refuse to look at the surgical incision or surgical dressing are having difficulty adjusting to the loss of a body part or with body disfigurement. This indicates the client is not yet ready to acknowledge the results of the surgery. All other options are expected.

An appropriate intervention for the patient during an acute attack of Meniere's Disease includes providing A. frequent respositioning B. quiet, darkened room C. television for diversion D. padded side rails on the bed

B

If a patient with blood type ORh+ is given AB RH- blood, what does the nurse wxpect to happen? A. Rh factor will react to the RBCs of the donor blood B. the anti-A and anti-B antibodies in the patient's blood will hemolyze the donor blood C. the anti-A and anti-B antibodies in the donor blood will hemolyze the patient's blood D. no adverse reaction will occur

B

Lab tests indicate increased fibrin split products. An appropriate nursing action is to monitor the patient for A. fever B. bleeding C. faintness D. thrombotic episodes

B

The patient was told she has carcinoma in situ, and the student nurse wonders what that is. How should the nurse explain this to the student nurse? a. evasion of the immune system by cancer cells b. lesion with histologic features of cancer except invasion c. capable of causing cellular alterations associated with cancer d. tumor cell surface antigens that stimulate immune response

B

What is the priority nursing intervention during the management of the patient with a pheochromocytoma? A. administering IV fluids B. monitoring the BP C. adminsitering beta blockers D. monitoring I/O

B

Which anemia is manifested with pancytopenia? A. thalassemia B. aplastic C. megaloblastic D. anemia of chronic disease

B

Which class of oral glucose-lowering agents is most commonly used for people with type 2 diabetes because it reduces hepatic glucose production and enhances tissue uptake of glucose? A. insulin b. biguanides c. meglitinide d. sulfonyurea

B

what is an appropriate nursing intervention for the patient with hyperparathyroidism? A. pad side rails as a seziure precaution B. increase fluid intake C. maintain bed rest to prevent fractures D. Monitor the patient for Troussau's and Chvostek's signs

B

when caring for the patient with nephrogenic DIm what should the nurse expect the treatment to include? A. fluid restriction B. thiazide diuretics C. high-sodium diet D. metformin

B

What characteristics describe unstable angina? (select all that apply) A. usually precipitated by exertion B. new-onset angina with minimal exertion C. occurs when person is recumbent D. characterized by increased duration or severity E. usually occurs in response to coronary artery spasm

B, D

which conditions characterize critical limb ischemia? A. cold feet B. arterial leg ulcers C. venous leg uclers D. gangrene E. no palpable peripheral pulses F. rest pain lasting more than 2 week

B, D, E

What characteristics of hearing loss are associated wit conductive loss (select all that apply) A. presbycusis B. speaks softly C. related to otitis media D. result of ototoxic drugs E. hears best in noisy environment F. may be caused by impacted cerumen

B,C,E,F

A patient with wet AMD is treated with photodynamic therapy. what does the nurse instruct the patient to do after the procedure? A. maintain the head in an upright position for 24 hours B. avoid blowing the nose or causing jerking movements of the head C. completely cover all the skin to avoid a thermal burn from sunlight D. expect to experience blind spots where the laser has caused retinal damage

C

A pt with a history of PUD is hospitalized with symptoms of a perforation. During the initial assessment, what should the nurse expect the patient to report? A. vomiting of bright-red blood B. projectile vomiting of undigested food C. sudden, severe generalized abdominal/back pain D. hyperactive bowel sounds and upper abdominal swelling

C

What lab finding is expected in ulcerative colitis as a result of diarrhea/vomiting? A. increased albumin B. elevated WBCs C. decreased serum Na, K, Mg, Cl, and HCO3 D. Decreased Hgb and HCT

C

What medication is used in hyperthyroidism to block the effects of the SNS? A. potassium iodine B. PTU C. Propranolol (inderal) D. radioactive iodine (RAI)

C

what can patients who are at risk for nephrolithiasis do to prevent kidney stones? A. lead an active lifestyle B. limit protein and acidic foods in the diet C. drink enough fluids to produce dilute urine D. take prophylactic antibodies to control UTIs

C

what type of urinary tract calculi are the most common and obstruct the ureter? A. cystine B. uric acid C. calcium oxalate D. calcium phosphate

C

A patient is scheduled for a CABG surgery. What does the nurse say is involved with the procedure? A. synthetic graft will be used as a tube for blood flow from the aorta to a coronary artery distal to an obstruction B. stenosed coronary artery will be resected, and a synthetic arterial tube graft will be inserted to replace the diseased artery C. the internal mammary artery will be detached rom the chest wall and attached to a coronary artery distal to the stenosis D. reversed segments of a saphenous artery from the aorta will be anastomosed to the coronary artery distal to an obstruction

C.

The nurse is planing to teach the patient with GERD about foods/beverages that decrease LES pressure. What should be included in this list? (select all that apply) A. alcohol B. root beer C. choclate D. citrus fruits E. fatty foods F. cola sodas

A, C, E, F

which statements describe anemia related to blood loss (select all that apply) A. a major concern is prevention of shock B. anemia is treated with intake of iron C. patient may manifest jaundice D. acute blood loss may lead to postural hypotension and incr HR E. initial clinical symptoms are the most reliable way to evaluate the effect and degree of blood loss

A,D,E

A patient with early cataracts tells the nurse that he is afraid cataract surgery may cause permanent visual damage. What should the nurse teach the patient? a. The cataracts will only worsen with time and should be removed as early as possible to prevent blindness. b. Cataract surgery is very safe and with the implantation of an intraocular lens, the need for glasses will be eliminated. c. Progression of the cataracts can be prevented by avoidance of ultraviolet (UV) light and good dietary management. d. Vision enhancement techniques may improve vision until surgery becomes an acceptable option to maintain desired activities.

D

After a patient had a renal arteriogram and is back on the clinical unit, what is the most important action by the nurse? A. observe for gross bleeding in the urine B. place the patient in High-Fowler's C. monitor the patients for signs of allergy to the contrast medium D. assess peripheral pulses in the involved leg every 30-60 minutes

D

The patient asks the nurse about valsartan (Diovan), the new medication prescribed for blood pressure. What is the best explanation the nurse can use to explain the action of this medication? A. prevents the conversion of angiotensin I to angiotensin II B. acts directly on smooth muscle of arterioles to cause vasodilation C. decreases exstracellular fluid volume by increasing Na and Cl excretion with water D. vasodilation, prevents the action of angiotensin II, and promotes increased salt and water excretion

D

to prevent airway obstruction in the postop patient who is unconscious or semiconscious, what will the nurse do? A. encourage deep breathing B. elevate the head of the bed C. adminsiter o2 per mask D. position the patient side-lying

D

what is a major method of preventing infection in the patient with neutropenia? A. prophylactic antibiotics B. a diet that eliminates fresh fruits/vegetables C. HEPA filtration D. strict handwashing

D

36 hours postop, a patient has a temperature of 100F. What is the most likely cause of this temperature elevation? A. dehydration B. wound infection C. lung congestion D. normal surgical stress response

D.

A client who has a history of myocardial infarction (MI) is prescribed aspirin (Ecotrin) 325 mg. The nurse correctly understands that the aspirin is ordered due to its action as an... analgesic. anti-inflammatory. antiplatelet aggregate. antipyretic.

antiplatelet aggregate. Aspirin is used to prevent blood from clotting. It also is used to reduce the risk of a second heart attack or stroke by reducing blood clotting in an artery, a vein, or the heart. Although aspirin does have an analgesic, anti-inflammatory and antipyretic effect, cardiac clients who take 325 mg daily are taking it for a different purpose.

A nurse is caring for a client following the surgical placement of a colostomy. Which of the following statements indicates the client understands the dietary teaching? "Eating yogurt can help decrease the amount of gas that I have. "I should eliminate pasta from my diet so that I don't have as many loose stools." "My largest meal of the day should be in the evening." "Carbonated beverages can help control odor."

"Eating yogurt can help decrease the amount of gas that I have." The client who has a colostomy can include yogurt into his diet to help reduce odors and intestinal gas. The client should also include pasta and other sources of fiber into his diet to help control loose stools. The client should have his largest meal in the middle of the day to help decrease the amount of stool produced during the hours of sleep. And the client should avoid carbonated beverages due to the increased production of intestinal gas.

A nurse is caring for a client who is using a patient-controlled analgesia pump for postoperative pain management. The nurse enters the room to find the client asleep and his spouse pressing the button to dispense another analgesia dose. Which of the following is an appropriate response for the nurse to make? "Next time you think he needs more medication, call me and I'll push the button." "It's a good idea to help make sure your husband can sleep comfortably." "Why do you think your husband needs more medication when he is asleep?" "We need to let your husband be the one who decides when he needs medication."

"We need to let your husband be the one who decides when he needs medication." To avoid overmedication, the client is the only one who should operate the PCA pump. In situations where the client is not able to do so, the provider may authorize a nurse or a family member to operate the pump.

A nurse is providing education for a client who has glaucoma. Which of the following statements is appropriate? "Without treatment glaucoma can cause blindness." "Double vision is a common symptom of glaucoma." "Glaucoma results from inadequate production of fluid within the eye." "You will need to treat glaucoma by instilling eye drops once a week."

"Without treatment glaucoma can cause blindness." The nurse should explain that without treatment glaucoma can result in blindness due to irreversible damage to the retina and optic nerve. The nurse should explain that a gradual loss of visual fields, rather than double vision, is a common symptom of glaucoma and is the result of increased intraocular pressure. The primary treatment for glaucoma includes the instillation of eye drops on an every 12 hr schedule.

A nurse is caring for a client who has cancer and is receiving palliative care. Which of the following statements by the client indicates they understand and accept their prognosis? "I am thinking of getting a second opinion." "I am hoping this will limit my discomfort." "This is making me stronger every time." "This is not working and I plan to stop treatment."

. "I am hoping this will limit my discomfort." Clients receiving palliative care are aware that the outcome is to prevent suffering and provide the best possible quality of life. They are aware that there is no hope for cure or recovery and the outcome is to maintain and support, not improve functional ability as they transition to other levels of care.

A nurse at an extended care facility is teaching a class of older adults about the expected physiologic changes of aging. Which of the following facts should the nurse include in the discussion? (Select all that apply.) Older adults have more difficulty seeing in low light or glare. Personality changes are common in older adults. Vital capacity of the lungs declines significantly between the ages of 20 and 70. Cardiac output decreases with age due to decreased heart size. Bladder capacity decreases and the kidneys become less efficient with age. Cognitive abilities decline after the age of 70.

1, 3 & 5. Older adults have more difficulty seeing in low light or glare, Vital capacity of the lungs declines significantly between the ages of 20 and 70 and Bladder capacity decreases and the kidneys become less efficient with age. Difficulty focusing close up may begin in the 40s, and the ability to distinguish fine details may begin to decline in the 70s. From 50 on, there is increased susceptibility to glare, greater difficulty in seeing at low levels of illumination, and more difficulty in detecting moving targets. With age, the brain loses some neurons and others become damaged. However, the brain adapts by increasing the number of synapses (connections between cells) and by regrowth in the dendrites and axons that carry messages in the brain. There is no concomitant change in personality associated with the changes in the brain. Personality is quite stable, and sudden changes may suggest disease processes. The maximum vital capacity of the lungs declines with age, approximately 10% each decade, probably due to both decreased elasticity and increased residual volumes. The heart will actually grow slightly larger with age. Maximal oxygen consumption during exercise declines by about 10% with each decade of adult life. However, cardiac output stays nearly the same because the heart pumps more efficiently. With age, the kidneys gradually become less efficient at extracting waste from the blood. Bladder capacity declines; however, urinary incontinence, which may occur after tissue atrophy, is not universal in the older adult.

A nurse is preparing to administer IV therapy to a client who is compromised. Which of the following are clinical manifestations of left-sided heart failure. (Select all that apply.) Dyspnea Gastrointestinal bloating Jugular vein distention Orthopnea Paroxysmal nocturnal dyspnea

1, 4 & 5. Dyspnea, Orthopnea and Paroxysmal nocturnal dyspnea Dyspnea, orthopnea and paroxysmal nocturnal dyspnea are clinical manifestations of left-sided heart failure. Gastrointestinal bloating and jugular vein distention are clinical manifestations of right-sided heart failure.

A nurse is caring for an older adult client who has left-sided heart failure. Which of the following assessment findings should the nurse expect? Frothy sputum Dependent edema Nocturnal polyuria Jugular distention

1. Frothy sputum Left-sided heart failure reduces cardiac output and raises pulmonary venous pressure. Manifestations include hacking cough, frothy sputum, wheezing, fatigue, and weakness. Dependent edema, Nocturnal polyuria and Jugular distention all indicate right-sided heart failure.

A nurse is reviewing the causes of osteoporosis with a group of nursing students. The nurse should include which of the following types of medication therapy as a risk factor for osteoporosis? Thyroid hormones Anticoagulants NSAIDS Cardiac glycosides

1. Thyroid hormones Long-term use of synthetic thyroid hormone, such as levothyroxine (Synthroid), can accelerate bone loss. Anticoagulants can cause blood loss, not bone loss. NSAIDs do not cause bone loss, although they can cause many other adverse effects, such as gastrointestinal bleeding. Cardiac glycosides can cause muscle weakness, but not bone loss.

A nurse is caring for a client with a compression fracture of a spinal vertebra. Just prior to an hour-long transport to the hospital, the client was medicated with intravenous morphine sulfate (Duramorph). On arrival, the neurosurgeon determines that urgent surgical intervention is indicated for the fracture. The nurse realizes that consent for the surgery... must be obtained from a relative of the client. can be inferred since the client consented to the transport. should be obtained from the client immediately. will be delayed until the morphine is metabolized.

1. must be obtained from a relative of the client. According to the case scenario, this client was given a narcotic that can alter his ability to understand within the subsequent 1 to 2 hours. Consequently, this client is not legally able to provide consent. Delaying consent until the morphine is metabolized could be dangerous to the client and may increase the chance of a life-long disability

A nurse is caring for a client who has a fractured right hip and a Jackson-Pratt (JP) drain in place after surgery for an open reduction and internal fixation. The nurse should understand that the purpose of this device is to... prevent fluid from accumulating in the wound. prevent bleeding from the surgical site. prevent the development of a wound infection. eliminate the need for wound irrigations.

1. prevent fluid from accumulating in the wound. The purpose of a JP drain is to promote healing by draining fluid from the wound. This prevents pooling of blood and fluid, which could contribute to discomfort, delay healing and provide a medium for infection. The JP drainage tube is threaded through the skin into the wound near the surgical incision and is held in place by sutures.

A nurse is caring for a client who is postoperative. Which of the following statements about pain management is true? (Select all that apply.) Administering medication for pain will eventually lead to addiction. Each person's expression of pain may be different and individualized. Intravenous narcotics are the longest acting pain relief method. Pain level and pain tolerance can be assessed using a scale from 0 to 10. The client will express the feeling of pain both verbally and nonverbally.

2 & 4. Each person's expression of pain may be different and individualized and Pain level and pain tolerance can be assessed using a scale from 0 to 10. Pain medication administered for the management of physical pain will not lead to addiction. Expression of pain is individualized and may differ. This is often influenced by the client's culture. Intravenous pain medication is the fastest acting. A scale of 0 to 10 is an effective method of monitoring a client's level of pain. The client may or may not be express the sensation of pain by both verbal and nonverbal means.

A nurse is caring for a client who is newly diagnosed with diabetes mellitus and is prescribed glipizide (Glucotrol). When instructing the client about this medication, the nurse should describe its method of action with which of the following statements? "Glucotrol absorbs the excess carbohydrates in your system." "Glucotrol stimulates your pancreas to release adequate insulin." "Glucotrol replaces insulin that is not being produced by your pancreas." "Glucotrol prevents your liver from destroying your insulin."

2. "Glucotrol stimulates your pancreas to release adequate insulin." Glipizide is an oral antidiabetic medication in the pharmacological classification of sulfonylurea agents. These medications help to lower blood glucose levels in clients who have type 2 diabetes mellitus using several methods, including reducing glucose output by the liver, increasing peripheral sensitivity to insulin, and stimulating the release of insulin from the beta cells of the pancreas.

A nurse is teaching a client with a new diagnosis of diabetes mellitus how to administer his insulin and to eat a healthy and balanced diet. The client tells the nurse that his brother, who also has diabetes, has taught him what foods he can and cannot eat. Which client statement indicates to the nurse the need for further teaching? "My brother told me to rotate my injection sites every day." "I know I have to eliminate bread and pasta from my diet." "I'll roll the insulin vial in my hands rather than shaking it." "I'll wash my feet every day but I won't soak them."

2. "I know I have to eliminate bread and pasta from my diet." Although the client will have to follow a balanced meal plan that limits the amount of carbohydrates he ingests at each meal and snack, there are no forbidden foods. The client can have bread and pasta in appropriate amounts as part of a balanced meal plan. The client should rotate injection sites to avoid the complication of lipodystrophy, which is an excess growth of fat beneath the skin. It reduces the absorption of insulin, thus reducing its effect. The client should roll the insulin vial to ensure that it is adequately suspended and to avoid the bubbles that shaking it can create. The client should wash his feet with warm water every dry, dry them well between the toes, avoid soaking them, and inspect them every day for any changes.

A nurse is caring for a client following cataract surgery. Which of the following comments made by the client should the nurse report to the client's provider? "My eye really itches, but I'm trying not to scratch it." "I need something for the pain in my eye. I can't stand it." "It's hard to see with one eye patched. I'm afraid of falling." "The bright light in this room is really bothering me."

2. "I need something for the pain in my eye. I can't stand it." Following cataract surgery, the client should be advised to immediately report any pain, decrease in vision, or increase in discharge in the eye. The nurse should know that eye pain after surgery may indicate acute angle glaucoma, which can cause permanent damage to the optic nerve if not treated promptly.

A nurse is teaching a client who has diabetes about which dietary source should provide the greatest percentage of her calories. Which of the following statements indicates the client understands the teaching? "Most of my calories each day should be from fats." "I should eat more calories from complex carbohydrates than anything else." "Simple sugars are needed more than other calorie sources." "Protein should be my main source of calories."

2. "I should eat more calories from complex carbohydrates than anything else." The client who has diabetes should consume the majority of her calories from complex carbohydrates such as whole grains, fruits, and vegetables. She should limit her intake of fats and simple sugars such as foods containing sucrose and should also consume 15 to 20% of her calories from protein sources.

A nurse is talking with a client who has peptic ulcer disease and is starting therapy with sucralfate (Carafate). The nurse should instruct the client to take the medication... with an antacid. 1 hr before meals. with food or milk. immediately after meals.

2. 1 hr before meals. Sucralfate is a mucosal protectant. The client should take it on an empty stomach, 1 hr before meals, for maximum effectiveness. The client should not take antacids within 30 min of taking sucralfate. And taking sucralfate with food or milk and on a full stomach reduces its effectiveness.

A nurse is giving discharge instructions to a client who had an internal repair of a right hip fracture. The nurse knows that teaching has been effective when the client tells her that he will rest during the day sitting on which of the following pieces of furniture? A reclining chair with an ottoman A straight-backed chair with an elevated seat A couch with plush cushions A rocking chair with a curved back

2. A straight-backed chair with an elevated seat A straight-backed chair with an elevated seat allows the client to assume proper positioning when sitting. An elevated seat decreases the risk of his hip dislocation. All other positions may increase the risk of hip dislocation.

In preparation for the discharge of a client with peripheral arterial disease (PAD), the nurse should include which of the following instructions? Apply a heating pad on a low setting to help relieve leg pain. Adjust the thermostat so that the environment is warm. Wear antiembolic stockings during the day. Rest with the legs above heart level.

2. Adjust the thermostat so that the environment is warm. The client's instructions should include keeping the environment warm to prevent vasoconstriction. Wearing gloves, warm clothes, and socks will help prevent vasoconstriction. Clients who have PAD should not apply heat directly to a limb because sensation is diminished and burns could result. Clients who have PAD should not wear any constrictive clothing. Extreme elevation of the legs can slow the flow of arterial blood to the feet.

A nurse is providing an education program on prostate cancer screening. Based on the level of risk, which of the following groups should the nurse educate first? Caucasian men African American men Asian men Hispanic men

2. African American men The greatest risk group is African American men; therefore, they are the first group the nurse should educate.

A nurse is discussing urinary tract infections (UTI) with a client. Which of the following should the nurse include as a risk factor? COPD. Diabetes mellitus. Anemia. Osteoporosis.

2. Diabetes mellitus. Diabetes mellitus is considered a risk for factor for a UTI due to the increased amount of glucose present in the urine.

Nursing responsibilities for the client with a patient-controlled analgesia (PCA) system should include: Reassuring the client that pain will be relieved. Documenting the client's response to pain medications on a routine basis. Instructing the client to continue pressing the system's button whenever pain occurs. Titrating the client's pain medication until the client is free from pain.

2. Documenting the client's response to pain medications on a routine basis. It is essential that the nurse document the client's response to pain medication on a routine, systematic basis. Reassuring the client that pain will be relieved is often not realistic. A client who continually presses the PCA button may not be getting adequate pain relief, but through careful assessment and documentation, the effectiveness of pain relief interventions can be evaluated and modified. Pain medication is not titrated until the client is free from pain but rather until an acceptable level of pain management is reached.

A home health nurse visits a client who has COPD and receives oxygen at 2 L/min via nasal cannula. The client tells the nurse she has been having difficulty breathing. Which of the following nursing actions is the priority at this time? Increase the oxygen flow to 3 L/min. Evaluate the client's respiratory status. Call emergency services for the client. Have the client cough and expectorate secretions.

2. Evaluate the client's respiratory status. The first action the nurse should take using the nursing process is to collect data from the client. The nurse should immediately evaluate the client's respiratory status before determining the appropriate interventions. All other options are correct, but not the priority.

A nurse is caring for a client who is scheduled to have surgery. In preparing the client for surgery, which of the following actions is considered outside the responsibility of the nurse? Assess the current health status of the client. Explain the operative procedure, risks, and benefits. Review preoperative laboratory tests results. Ensure that a signed surgical consent form was completed.

2. Explain the operative procedure, risks, and benefits. Explaining the procedure and any risks that may be associated with the procedure is the responsibility of the person performing the procedure. This is not a nursing responsibility.

A client complains of shortness of breath and chest pain the first day following multiple long bone fractures. The nurse would consider which of the following client complications when assessing the client? Pneumonia Fat emboli Cardiac dysrhythmia Hypoxic condition

2. Fat emboli The client with a compound long bone fracture is at high risk for developing a fat embolus within 24 to 96 hours, which may lead to cardiac dysrhythmia or a hypoxic condition.

A nurse is caring for a client who has benign prostatic hyperplasia (BPH). Which of the following medications should the nurse plan to administer? Danazol (Cyclomen). Finasteride (Proscar). Fluoxymesterone (Androxy). Methyltestosterone (Android).

2. Finasteride (Proscar). Proscar, a 5-alpha-reductase inhibitor, is used in the treatment of BPH to prevent the conversion of testosterone and to decrease prostate size. Cyclomen is an anabolic steroid used in the treatment of endometriosis. Androxy and Android, anabolic steroids, are contraindicated for clients who have BPH.

A nurse is planning care for a client who has metastatic breast cancer and is being treated with chemotherapy and radiation. Admission laboratory results indicate that the client has neutropenia. The nurse should understand that which of the following should be indicated as a restriction in the client's plan of care? All visitors from entering the client's room. Fresh fruits and vegetables in the diet. Oral fluid intake to between meals only. Activities that could result in bleeding.

2. Fresh fruits and vegetables in the diet. Clients who are receiving chemotherapy and radiation therapy are likely to become immunocompromised as a result of neutropenia, a decreased white blood cell (WBC) count. Because bacteria on produce are not always washed off easily, immunocompromised clients are instructed not to eat fresh fruits and vegetables. In addition, the client is instructed to eat only well-cooked meat. Immunocompromised clients are more susceptible to infection and illness from food-borne bacteria than other clients.

A client is taking ibuprofen (Advil, Motrin) to treat hip pain. The nurse should teach the client that, to minimize gastric mucosal irritation, she should take this medication at which of the following times? At bedtime. Immediately after a meal. On arising. On an empty stomach.

2. Immediately after a meal. To minimize gastric irritation, the client should take ibuprofen with a meal or immediately after a meal. All other options increase the risk of gastric irritation.

A nurse is preparing to administer a prescribed dose of 10 units of regular insulin and 20 units of NPH insulin to a client who has diabetes. The nurse should take which of the following actions first when mixing the two types of insulin? Inject 10 units of air into the regular insulin bottle. Inject 20 units of air into the NPH insulin bottle. Withdraw 10 units of air from the regular insulin bottle. Withdraw 20 units of air from the NPH insulin bottle

2. Inject 20 units of air into the NPH insulin bottle. Injecting 20 units of air into the NPH insulin bottle is correct. The first action the nurse should take is to inject 20 units of air into the NPH insulin bottle, because this is the intermediate-acting insulin. All other options are correct, but they are not the first actions the nurse will take.

A nurse is preparing to administer insulin lispro (Humalog) to a client who has type 1 diabetes mellitus. Which of the following nursing actions is appropriate? Assess for hypoglycemia 4 hr after the insulin injection. Inject the insulin 15 min before a meal. Monitor for polyuria. Administer with short-acting insulin.

2. Inject the insulin 15 min before a meal. The appropriate nursing action is to administer the insulin 15 min before a meal because insulin lispro is a rapid-acting insulin and the client may develop hypoglycemia quickly if they don't eat. The appropriate nursing action is to assess for hypoglycemia from 1 to 3 hr not 4 hr after the insulin injection. The appropriate nursing action is not to monitor for polyuria after the injection. Polyuria is a symptom of hyperglycemia.The appropriate nursing action is that insulin lispro may be given with other longer- acting insulin in the same syringe.

A nurse at a provider's office receives a phone call from a client who reports unrelieved chest pain after taking a nitroglycerin (Nitrostat) tablet 5 minutes ago. Which of the following is an appropriate response by the nurse? Tell the client to take an aspirin. Instruct the client to call 911. Have the client take another nitroglycerin tablet in 15 min. Advise the client to come to office.

2. Instruct the client to call 911. The standard dosing regimen for nitroglycerin is one tablet sublingually. If anginal pain is not relieved after 5 min, the client should call 911 or go to the emergency department. An additional two tablets can be taken at 5 min intervals while awaiting emergency care. While taking an aspirin may not cause additional harm, with unrelieved chest pain this is not an appropriate response by the nurse. Having the client take another nitroglycerin tablet in 15 min or advising the client to come into the office delays needed treatment for the client, and is not appropriate.

A nurse is caring for a client who has undergone a transurethral resection of the prostate (TURP) to correct benign prostatic hypertrophy (BPH) and has returned to the unit from the post anesthesia care unit with an indwelling urinary catheter. For several hours, the client's urinary output is adequate. After 5 hr, the nurse notes that the catheter has drained no urine during the prior hour. Which of the following actions should the nurse take first? Increase the client's oral fluid intake. Irrigate the catheter. Request an increased intravenous fluid rate. Monitor the urinary output frequently.

2. Irrigate the catheter. The most common procedure for surgical removal of the prostate for BPH is the TURP. A urinary catheter is placed to drain the bladder after surgery. A sudden stop in urine output from the catheter following prostate surgery is likely a result of an occluded catheter. Irrigation will remove a clot from the catheter.

A staff nurse is working on a medical-surgical floor for a client who has Addison's disease. The client asks the nurse what causes Addison's disease. Which of the following is an appropriate response by the nurse? Overproduction of insulin by the pancreas. Lack of production of cortisol by the adrenal gland. Overproduction of growth hormone by the pituitary gland. Lack of production of lymphocytes by the thymus gland.

2. Lack of production of cortisol by the adrenal gland. Addison's disease is caused by a lack of production of cortisol by the adrenal gland.

A nurse is caring for a client who has type 1 diabetes mellitus. The nurse misread the client's morning blood glucose level as 210 mg/dL instead of 120 mg/dL. Based on this error, she administered the insulin dose appropriate for a reading over 200 mg/dL before the client's breakfast. Which of the following is the nurse's priority action upon realizing the error? Give the client 15 to 20 g of carbohydrate. Monitor the client for hypoglycemia. Complete an incident report. Notify the nurse manager.

2. Monitor the client for hypoglycemia. The first action the nurse should take using the nursing process is to assess or collect data from the client. The nurse should immediately check the client's blood glucose level, expecting it to be low because of the unnecessary dose of insulin. If it is within the expected reference range, the nurse should continue to monitor the client for hypoglycemia. It might become necessary to administer a ready source of carbohydrate to counteract the effects of the unnecessary dose of inulin, but this is not the nurse's immediate priority at this time. The nurse will have to complete an incident report detailing the medication error, but this is not the nurse's immediate priority at this time. The nurse will have to notify the nurse manager about the medication error, but this is not the nurse's immediate priority at this time.

A nurse is caring for an older adult client who was alert and oriented. After four days of hospitalization, the client seems to be increasingly restless and intermittently confused. What is the most appropriate nursing intervention to address the safety needs of the client? Call the family and ask them to stay with the client. Move the client to a room closer to the nurse's station. Apply wrist and leg restraints to the client. Administer medication to sedate the client.

2. Move the client to a room closer to the nurse's station. This option is the most appropriate intervention to address the safety needs of the client. By moving the client to a room closer to the nurse's station, it will be easier for the staff to observe the client should the client behave in an unsafe or inappropriate manner. It is the nurse's responsibility to maintain the client's safety while the client is hospitalized, not the family's. There are other measures that take priority over restraint and would be more effective at assuring the client's safety. Sedating an older adult client often makes the client more confused and more likely to behave inappropriately.

A nurse is caring for a client who has diabetes and reports an occasional loss of sensation in her feet. Which of the following complications is the client likely experiencing? Nephropathy Neuropathy Macroangiopathy Retinopathy

2. Neuropathy Neuropathy is loss of sensation, and is likely the complication this client is experiencing. Nephropathy is kidney dysfunction, macroangiopathy is arteriosclerosis, and retinopathy is an eye complication.

A nurse gives a client morphine sulfate 2 mg IV push after the client reports pain. The nurse evaluates the client 15 min after the injection. Which of the following findings represent an adverse effect? Sleepy but arousing when name is called. Respiratory rate of 8 breaths per minute. Numerical scale for pain reported from a level of 6 to 4. Pulse oximeter reading of SaO2 equal to 94%.

2. Respiratory rate of 8 breaths per minute. The nurse's evaluation of the client's displaying respiratory depression of 8 per minutes represents an adverse effect of the morphine. All other options are not adverse effects of morphine.

A nurse is providing discharge instructions for a client who has undergone outpatient cataract surgery with insertion of an intraocular lens. Which of the following should the nurse include? For discomfort take aspirin. Restrict lifting objects greater than ten pounds. Reduction of vision after surgery is normal. Apply warm compresses for discomfort.

2. Restrict lifting objects greater than ten pounds. The information the nurse should include is to restrict lifting objects greater than ten pounds, which may cause an increase in intraocular pressure. The nurse should instruct the client to take acetaminophen, not aspirin, for discomfort. Aspirin inhibits platelet aggregation which may increase bleeding. A reduction of vision after surgery is not normal and the client needs to notify the surgeon immediately. The nurse should instruct the client to apply cool, not warm, compress for discomfort.

A nurse is caring for a client who is one week postoperative following abdominal surgery. While changing the client's abdominal dressing the nurse notes the presence of serosanguineous drainage. The nurse should recognize which of the following? Serosanguineous drainage at this time is expected after abdominal surgery. Serosanguineous drainage at this time is a manifestation of possible dehiscence. Serosanguineous drainage at this time is a manifestation of hemorrhage. Serosanguineous drainage at this time is a manifestation of infection.

2. Serosanguineous drainage at this time is a manifestation of possible dehiscence. Serosanguineous drainage beyond the fifth postoperative is a manifestation of possible dehiscence and the provider should be notified. Serosanguineous drainage is not expected one week after abdominal surgery. Yellow crusting and a pink color to the incision line are expected at this time frame. Serosanguineous drainage is not a manifestation of hemorrhage or infection. Serosanguineous drainage is serum-like or yellow in color, instead of the thick, yellow pus considered a manifestation of infection.

A nurse is discussing the care of a client who has type 1 diabetes mellitus with the assistive personnel (AP). Which of the following situations should the nurse instruct the AP to report immediately? The client reports dizziness when standing. The client refuses breakfast and requests to sleep. The client asks the AP to trim his broken toenail. The client reports urine that is dark yellow in color.

2. The client refuses breakfast and requests to sleep. A client with diabetes mellitus type 1 will typically be receiving insulin just before breakfast, and missing a breakfast meal means that the client may experience hypoglycemia later in the morning. This situation needs to be reported to the nurse immediately. It is not unusual for a client with chronic disease to experience transient dizziness when changing positions, due to orthostatic hypotension. It is potentially dangerous for a client who has diabetes to have his toenails clipped by a person who is not specially trained because of the possibility of infection and impaired peripheral circulation. However, this is not a situation that needs to be reported immediately. Dark yellow urine is not an immediate concern. It may indicate mild dehydration, which will resolve with additional fluid intake.

A nurse is planning care for several clients and the nurse is considering the client's risk for ischemic embolic stroke. Which of the following clients are at risk? The client who has uncontrolled hypertension. The client who has chronic atrial fibrillation. The client who has thrombocytopenia The client who has an arteriovenous malformation.

2. The client who has chronic atrial fibrillation. Chronic atrial fibrillation places the client at risk for embolic stroke because a small thrombus may dislodge and migrate to the brain. Uncontrolled hypertension places the client at risk for hemorrhagic stroke. Thrombocytopenia places the client at risk for bleeding-induced stroke. Arteriovenous malformation places the client at risk for hemorrhagic stroke.

A client who is 1 day postoperative following a thyroidectomy reports severe muscle spasms of the lower extremities. Which action should the nurse implement? Check the pedal pulses. Verify the most recent calcium level. Request medical order for relaxant. Administer an oral potassium supplement.

2. Verify the most recent calcium level. A client who has had a thyroidectomy is at risk of hypocalcemia due to possible disruption of the parathyroid gland during surgery. The parathyroid glands are four small glands located inside the thyroid gland. They are responsible for calcium regulation; if they are damaged during a thyroidectomy, there is a risk of hypocalcemia. Low calcium levels can be manifested as numbness and tingling of the fingers and around the mouth, muscle spasms (particularly of the hands and feet), and hyperactive reflexes. If a client develops any of these symptoms following a thyroidectomy, it would be important to see what the latest calcium level is. The expected reference range for calcium is 8.5-10.5 mg/dL. If a calcium level is not available, one should be ordered. If the calcium is low, the provider should be notified, and oral or intravenous calcium replacement should be administered.

A nurse is discontinuing a course of prednisone (Deltasone) for a client with an exacerbation of asthma. The nurse should taper the dose so that the client does not experience... hyperglycemia. adrenocortical insufficiency. severe dehydration. rebound pulmonary congestion.

2. adrenocortical insufficiency. Prednisone, a corticosteroid, is similar to cortisol, the glucocorticoid hormone produced by the adrenal glands. It relieves inflammation and is used to treat certain forms of arthritis, severe allergies, autoimmune disorders, and asthma. Administration of glucocorticoids may depress the body's normal adrenocortical activity, and abrupt withdrawal of the drug can lead to a syndrome of adrenal insufficiency.

A nurse is assessing a client who has a long history of smoking and is suspected of having laryngeal cancer. The nurse should anticipate that the client will report that her earliest manifestation was... dysphagia. hoarseness. dyspnea. weight loss.

2. hoarseness. Laryngeal cancer, a malignant tumor of the larynx, is most often caused by long exposure to tobacco and alcohol. Hoarseness that does not resolve for several weeks is the earliest manifestation of cancer of the larynx because the tumor impedes the action of the vocal cords during speech. The voice may sound harsh and lower in pitch than normal. All other options are later manifestations.

A nurse is obtaining informed consent from a client prior to surgery. Which of the following is necessary for informed consent to be valid? (Select all that apply.) A client's ability to pay for the consented surgical procedure. A client's ability to read the consent form. Disclosure of the treatment is provided. Client understands the surgical procedure. Voluntary consent is given.

3, 4 & 5. Disclosure of the treatment is provided, Client understands the surgical procedure and Voluntary consent is given. The client's ability to pay for the consented surgical procedure is not related to informed consent. It is also not necessary for the client to personally read the consent form. The client should be informed of treatment that is to be provided as well as the risks. The client should understand the surgical procedure as well as the risks. Informed consent protects the client, provider, and institution/employees. The client should give voluntary consent without influence for the procedure. Informed consent protects the client, provider, and institution/employees.

A nurse is caring for an older adult client whose provider will discharge him to an extended-care nursing facility the following morning. The client asks the nurse why he has to go to "that place." Which of the following is an appropriate nursing response? "Your doctor feels that this is the best place for you right now." "Why don't you ask your doctor about that when he comes in to see you?" "Did your doctor or anyone else talk to you about going to the nursing home?" "Your family can't take care of you at home, so you will need to go there."

3. "Did your doctor or anyone else talk to you about going to the nursing home?" It is important to identify what the client thinks he has heard about his discharge. Clarification of information can proceed after this.

A nurse is caring for a client who is receiving esomeprazole (Nexium) to manage GERD. Which of the following best indicates the desired therapeutic effect? "I don't pass gas as often." "My abdomen is no longer firm." "I don't have pain in my stomach. "I have regular bowel movements."

3. "I don't have pain in my stomach." Esomeprazole (Nexium) is a proton pump inhibitor (PPI) and works in the parietal cells of the stomach by inhibiting the proton pump enzyme that generates gastric acid secretion. It is used in the treatment of gastric ulcers, duodenal ulcers and GERD. An expected finding of the medication will be a decrease in the client's symptoms of an ulcer or GERD. Heartburn is a common sign of GERD, so absence of pain would be an indication that the medication is working. PPIs are generally well-tolerated.

A nurse is providing discharge instructions to a client who has congestive heart failure. Which of the following statements by the client indicates to the nurse that the teaching was effective? "I will do all of my activities in the morning and rest every afternoon." "I should attempt to exercise every day of the week." "I plan to slow down if I am tired the day after exercising." "I will take my diuretic before sleep and drink fluids during the day."

3. "I plan to slow down if I am tired the day after exercising." Clients who experience chest pain or dyspnea while exercising or experience fatigue the next day are probably advancing the activity too quickly and should slow down. Congestive heart failure results in decreased cardiac output. Clients should be encouraged to stay as active as possible without overdoing it. It might not be realistic for the client to do every activity in the morning without feeling tired, in which case the client should space out physical activity during the day. Clients with decreased cardiac output may not have the physical stamina to exercise on a daily basis. Therefore, the nurse should encourage the client to begin walking 200 to 400 feet per day. When at home, encourage the client to walk at least three times per week. The client should slowly increase the amount of time walked (perhaps 10 minutes a week) over several months. Clients should be advised to take diuretics in the morning to avoid waking during the night for voiding.

A nurse is administering an oral medication to an older adult client. The client states, "The pill I always take is green. I don't take an orange pill." Which of the following nursing responses is appropriate? "Sometimes the same pill comes in a different color." "Let me explain the purpose of the medication." "I will check your medication orders again." "This is the medication that your doctor wants you to take."

3. "I will check your medication orders again." The appropriate nursing response is to check the provider's original medication order to avoid a medication error.

A nurse is caring for a client who is taking ferrous sulfate (Feosol) tablets twice daily for iron-deficiency anemia. The client asks the nurse why the provider suggested that she take the ferrous sulfate with orange juice. Which of the following is an appropriate response by the nurse? "The orange juice will help you avoid becoming constipated." "The medication has an unpleasant taste, and the orange juice will help to disguise it." "The orange juice will help you absorb the medication more efficiently." "The medication can cause nausea, and the orange juice will prevent this."

3. "The orange juice will help you absorb the medication more efficiently." Ferrous sulfate provides the iron needed by the body to produce red blood cells. Taking iron supplements with a citrus fruit juice, such as orange juice, helps to increase the bioavailability of the iron.

A nurse is caring for a client who is scheduled for cardiac surgery, has an advance directive health care proxy form, and asks for clarification regarding the form. Which of the following responses by the client should indicate a need for clarification? "I can cancel or change my health care proxy at any time." "My health care and end-of-life choices will be made by my proxy." "The person designated as my health care proxy is legally obligated to abide by the wishes set forth in my living will." "The health care proxy does not go into effect until I am incapable of making decisions."

3. "The person designated as my health care proxy is legally obligated to abide by the wishes set forth in my living will." An advance directive is not a legal document. The health care proxy is morally bound to follow the wishes of the client, not legally. All the other options are correct statements regarding a health care proxy.

A nurse is caring for an older adult client whom is suspected of being neglected. Which of the following statements by the caregiver should be of most concern to the nurse? "We only have enough money for two meals a day." "We sit outside every afternoon." "We buy the prescriptions we can afford." "We cannot afford two new batteries for his hearing aid."

3. "We buy the prescriptions we can afford." This statement should be of greatest concern to the nurse. It reflects that not all prescriptions are being given to the older adult. Further information needs to be acquired to determine which medications are being given and which prescriptions are not being filled. A referral to social services can assist the client and family with purchasing prescriptions. In addition, the nurse should educate the client and family on the importance of correct medication administration. Although the nurse would want to inquire further about the other statements, they are not of most concern.

A nurse is admitting a client who has acute heart failure following myocardial infarction (MI) and is reviewing the provider's orders. Which of the following prescriptions by the provider requires clarification? Morphine sulfate 2 mg IV bolus every 2 hr PRN pain Laboratory testing of serum potassium upon admission 0.9% normal saline IV at 50 mL/hr continuous PT and PTT Bumetanide (Bumex) 1 mg IV bolus every 12 hr

3. 0.9% normal saline IV at 50 mL/hr continuous 0.9% sodium chloride is isotonic and will not cause the fluid shift needed in this client to reduce circulatory overload. Morphine sulfate is administered to clients with acute heart failure to reduce anxiety, decrease cardiac preload and afterload, slow respirations, and manage pain associated with MI. Serum potassium is monitored due to the use of diuretic therapy in a client with acute heart failure. Bumex, a loop diuretic, is administered as a first-line drug of choice for the client in acute heart failure.

A nurse is assessing a client who shows signs of hypoxemia during an asthma attack. Which of the following manifestations should the nurse expect? Nausea Dysphagia Agitation Hypotension

3. Agitation The nurse should expect agitation due to neurological changes from poor oxygen exchange. The nurse would not expect the client to be nauseated during an asthma attack. The client would display dyspnea not dysphagia, during an asthma attack. And the nurse should expect hypertension due to increase work load of the heart from decreased oxygenation.

A nurse is caring for a client who has cancer and is receiving total parenteral nutrition (TPN). Which of the following lab values indicates the treatment is effective? Hct 43% WBC 8,000/uL Albumin 4.2 g/dL Calcium 9.4 mg/dL

3. Albumin 4.2 g/dL Clients who have cancer can receive TPN to provide needed proteins and glucose they are otherwise unable to obtain. An albumin level of 4.2 g/dL is within the expected reference range and indicates the client is receiving adequate amounts of protein.

A nurse is caring for a client following arthroscopic knee surgery. To prevent postoperative complications, the nurse should have the client do which of the following? Remain on bedrest for the first 24 hrs. Keep the leg in a dependent position. Apply ice to the affected area. Begin active range of motion.

3. Apply ice to the affected area. Arthroscopy is a surgical procedure used to visualize, diagnose and treat problems inside a joint. Applying ice to the affected area in the immediate postoperative period reduces pain and swelling. When the client has recovered from sedation, the client will be allowed to walk, as tolerated, but should be instructed not to overuse or strain the joint for a few days. Elevating the affected area in the postoperative period reduces pain and swelling. Although the client will be allowed to walk as tolerated, joint use should be minimized for the first few days to reduce postoperative pain and swelling.

A nurse is teaching a client who is at risk for iron-deficiency anemia about optimizing her dietary intake of iron. The nurse should explain that which of the following sources of iron is easiest for the body to absorb? Spinach Cantaloupe Chicken Lentils

3. Chicken Food sources of iron fall into two categories - heme iron (from lean red meat, poultry, and fish) and nonheme iron (from fruit, vegetables, grains, and dried peas and beans). The body more easily absorbs heme iron.

A nurse is caring for a client who has a long history of peptic ulcers and is admitted for treatment of pyloric obstruction. The nurse is preparing to insert a nasogastric tube. Which of the following options is the rationale for the use of the nasogastric tube? Determine the pH of the gastric secretions Supply nutrients via tube feedings Decompress the stomach Administer medications

3. Decompress the stomach Due to a pyloric obstruction, removal of gastric secretions and gas from the stomach is needed. This is the purpose of the nasogastric tube.

A nurse is preparing a teaching plan for a client who has just found out that she has type 2 diabetes mellitus. What is the nurse's priority in preparing this plan? Establish short-term, realistic goals for the client. Give her access to a video about diabetes Determine what the client knows about managing her diabetes Evaluate the effectiveness of the client's admission teaching plan

3. Determine what the client knows about managing her diabetes The first action the nurse should take using the nursing process is to assess or collect data from the client. The nurse should find out what the client knows before proceeding with the plan.

A nurse is caring for a client who is postoperative following vascular surgery. Which of the following signs should indicate to the nurse that the client has developed a thrombus? Positive Kernig's sign. Positive Homan's sign. Dull, aching calf pain. Soft, pliable calf muscle.

3. Dull, aching calf pain. Dull, aching calf pain is a sign of deep-vein thrombosis. Other manifestations are edema, warmth, and redness in the calf. Kernig's sign indicates meningeal irritation. Homan's sign is unreliable as only a small percentage of clients who have a thrombus develop it, and performing it could mobilize the clot. A thrombus is more likely to cause muscle rigidity than a soft and pliable muscle.

A client comes to the emergency department via ambulance to report severe radiating chest pain and shortness of breath. The client appears restless, frightened, and slightly cyanotic. The provider prescribes oxygen by nasal cannula at 4 L/min stat, cardiac enzyme levels, IV fluids, and a 12-lead ECG. Which of the following actions should the nurse take first? Attach the leads for a 12-lead ECG Obtain the blood sample. Initiate oxygen therapy. Begin IV fluid infusion.

3. Initiate oxygen therapy. The greatest risk to the client's safety is myocardial ischemia. Thus the priority is to administer oxygen to help minimize this. It is important to determine the client's heart rhythm and cardiac enzyme levels and to begin the IV fluid infusion, but these are not the highest priorities at this time.

A nurse is reinforcing discharge teaching with an older adult client. Which of the following should be the nurse's priority action? Ask the client to verbalize understanding of previous teaching. Observe ability to access information from the Internet. Minimize distractions by closing the door to the room. Present learning activities from simple to complex tasks.

3. Minimize distractions by closing the door to the room. The nurse should initially ensure the room is private, free from distraction, and well lit. Approximately one fifth of the adults in the United States older than 70 have vision and hearing impairment. It is imperative to create an environment conducive to learning. This is the most appropriate action for the nurse to take prior to beginning communication.

A nurse is caring for a client who is on a 2,000-calorie American Diabetes Association (ADA) diet and substitutes the whole milk on his breakfast tray with skim milk. Because of this substitution, the nurse should know that the client can add which of the following items to the oatmeal on his breakfast tray? One 1/8 teaspoon salt. One ounce of raisins. One pat of butter. One teaspoon of brown sugar.

3. One pat of butter. Substituting skim (fat-free) milk for whole milk allows the client to add a fat exchange to his breakfast tray. A fat exchange usually varies in serving size, but one pat of butter (about 1 tsp) represents one fat exchange.

A nurse is planning care for a client who has a suspected myocardial infarction. Which of the following should the nurse administer first? Nitroglycerin (Nitrostat) Aspirin (A.S.A.) Oxygen Morphine Sulfate

3. Oxygen The priority action the nurse should take when using the airway, breathing, circulation (ABC) approach to client care is to apply oxygen.

A nurse is caring for a client who has hyperparathyroidism. Based on this diagnosis, the nurse is aware that the client is at risk for which of the following? Impaired skin integrity Fluid retention Pathologic fractures Dysphagia

3. Pathologic fractures Hyperparathyroidism results in the release of calcium and phosphate into the blood, thereby decreasing bone density. This places the client at risk for pathologic fractures. The client is not at risk for impaired skin integrity, fluid retention or dysphagia.

A nurse is caring for a client who enters the emergency department of severe chest pain. Which of the following interventions should the nurse implement to determine if the client is experiencing a myocardial infarction? Check the client's blood pressure. Auscultate heart tones. Perform a 12-lead ECG Determine if pain radiates to the left arm.

3. Perform a 12-lead ECG The nurse should perform a 12-lead ECG when a client complains of chest pain to determine if the client is experiencing a myocardial infarction. The nurse should check the client's vital signs and auscultate heart tones when chest pain is present, however, these findings will not determine if the client is experiencing a myocardial infarction. The nurse should also identify the location of pain as part of a complete assessment, however radiation to the left arm can be present in other conditions and therefore does not indicate that the client is experiencing a myocardial infarction.

A nurse in the post anesthesia care unit is assessing a client who has a colostomy after a colectomy. Which of the following conditions should the nurse report to the provider? Stoma oozing red drainage Shiny moist stoma Purplish colored stoma Rosebud appearing stoma orifice

3. Purplish colored stoma The client whose stoma is purplish in color indicates ischemia and the provider should be notified immediately.

A client who is 2 days postoperative following abdominal surgery is about to progress from a clear liquid diet to full liquids. Which of the following items should the nurse tell the client he may now request to have on his meal tray? Cranberry juice Flavored gelatin Skim milk Chicken broth

3. Skim milk Full liquids include milk and milk products, so the client may now ask for skim milk. All other options are a liquid diet.

A nurse is monitoring a client's transfusion of packed red blood cells and suspects that a hemolytic reaction is occurring. Which of the following is the priority intervention? Assess the client's respiratory rate. Administer 0.9% sodium chloride through the IV line. Stop the transfusion. Notify the blood bank.

3. Stop the transfusion. When suspecting a hemolytic reaction the priority action by the nurse is to immediately stop the transfusion to prevent further hemolysis.

A client who has just had abdominal surgery returns to the unit from the post anesthesia care unit with an IV fluid infusion and an NG tube in place. Which of the following is the assessment priority for the nurse who is caring for the client? The IV catheter insertion site. The level of the client's pain. The surgical dressing. The patency of the NG tube.

3. The surgical dressing Although priorities may shift with the type of surgery and the client's postoperative status, in general, the greatest risk to the client's safety of these options immediately after surgery is hemorrhage. Thus the nurse's priority assessment is the surgical dressing for bleeding

A nurse is caring for an older adult client who is hospitalized. At bedtime, the client says, "I am afraid that I may fall while walking to the bathroom during the night. I tend to get a bit disoriented in new surroundings." The nurse should: limit the client's fluid intake in the evening. obtain a bedside commode for the client's use. leave a night-light on in the client's room. put the side rails up and tell the client to call the nurse before voiding.

3. leave a night-light on in the client's room. This is an appropriate action for keeping the client safe. Night vision may be impaired in older adult clients. If the client awakens in the night, a night-light may help the client to recognize the surroundings and decrease the likelihood of disorientation. It will also help to decrease the possibility of a fall on the way to the bathroom because the path will be illuminated and the client will be less likely to trip over objects in the room.

A nurse is discharge teaching a client who has GERD. Which of the following client statements reveals an understanding of the teaching? "I can eat whatever I want." "I will sleep on my left side." "I will lie down following meals." "I will sleep with the head of my bed elevated."

4. "I will sleep with the head of my bed elevated." The client should sleep with the head of the bed elevated 6 to 12 inches for sleep to prevent reflux at night. The client should avoid spicy and acidic foods, caffeine, and carbonated beverages. The client should lie on the right side position to decrease symptoms of night time reflux. The client should sit up in a chair following meals to decrease reflux.

A nurse is presenting discharge instructions to a client who has multiple sclerosis (MS). The client reports symptoms of diplopia, dysmetria, and sensory change. Which of the following nursing statements are appropriate? "Wear an eye patch on the right eye at all times." "Plan to relax in a hot tub spa each day." "Engage in a vigorous exercise program." "Implement a schedule to include periods of rest."

4. "Implement a schedule to include periods of rest." The nurse should implement a schedule with periods of exercise followed by periods of rest to maintain muscle strength and coordination. The nurse should instruct the client to alternate every two hours an eye patch to improve diplopia, not leave on the right eye continually, to avoid extreme temperature changes which may exacerbate the MS symptoms and to develop a tolerable exercise program, not a vigorous exercise program, which may exacerbate the MS symptoms.

A home health care nurse is visiting an older adult client who tells the nurse that she is feeling tired and is unable to shop for groceries. The client asks the nurse if she could pick up a few things for her at a local store. Which of the following is an appropriate nursing response? "It is against policy for me to grocery shop for you." "I would be happy to do whatever I can to help you." "What I think you should do is wait for the days when you feel better and do your grocery shopping then." "It's unfortunate that you don't have someone to help you on the days you don't feel well. Let's talk about how we can solve that for you."

4. "It's unfortunate that you don't have someone to help you on the days you don't feel well. Let's talk about how we can solve that for you." Acknowledging that the client needs assistance on certain days and offering to find a solution for that problem are appropriate nursing responses. Part of the role of nurses in home care is to function according to clients' needs by providing what they require, coordinating their care, or making the appropriate referrals to community or other resources.

A provider diagnoses a client with cancer and advises immediate chemotherapy. The client tells the nurse that she wants to try nontraditional treatments first. Which of the following is an appropriate nursing response? "Using nontraditional treatments is not a good Idea. I'd rather you avoid that route." "A lot of people think nontraditional treatments will work, and they find out too late that they made the wrong choice." "Your doctor is very knowledgeable. If he prescribes chemotherapy, it's the best treatment for you." "Tell me more about your concerns about taking chemotherapy."

4. "Tell me more about your concerns about taking chemotherapy." Asking the client to talk more about her fears and her concerns encourages communication. It is an example of the therapeutic communication technique of exploring. All other options are non-therapeutic.

A nurse provides health screening education to a group of clients. Which of the following clients has the greatest risk for hypertension? A 56-year old Chinese male. A 62-year-old Jewish female A 53-year-old Spanish female A 65-year-old African American male

4. A 65-year-old African American male The incidence of hypertension is higher in African Americans than in European Americans. African Americans tend to develop more severe hypertension at an earlier age and have twice the risk for complications, such as stroke and heart attack. Hypertension-related deaths are also higher among African Americans. An adult's blood pressure tends to rise with advancing age. Older adults often have a rise in systolic pressure related to decreased vessel elasticity. Blood pressure greater than 140/90 mm Hg is defined as hypertension and increases an older adult's risk for hypertension-related illnesses.

A nurse in an urgent care center is caring for a client who is having an acute asthma exacerbation. Which of the following actions is the nurse's highest priority? Initiating oxygen therapy Providing immediate rest for the client Positioning the client in high-Fowler's Administering a nebulized beta-adrenergic

4. Administering a nebulized beta-adrenergic The greatest risk to the client's safety is airway obstruction. Beta-adrenergic medications decrease the inflammatory response that triggers narrowing of the airways. They provide prompt relief of airflow obstruction and are the initial intervention when a client has an acute asthma exacerbation. All the other options are correct, but they are not the highest priority for a client having an acute asthma exacerbation.

A nurse is teaching a client has a new diagnosis of aplastic anemia. When discussing the pathology of this disease, which of the following is appropriate to include in the teaching? Aplastic anemia is associated with the decreased intake of iron. Aplastic anemia results in an increased rate of RBC destruction. Aplastic anemia is directly related to impaired liver function. Aplastic anemia results from decreased bone marrow production of RBC.

4. Aplastic anemia results from decreased bone marrow production of RBC. Aplastic anemia is a hypoproliferative anemia resulting from decreased production of RBC within the bone marrow. An inadequate intake of iron can result in iron deficiency anemia rather than aplastic anemia. Hemolytic anemia, rather than aplastic anemia, is associated with the increased rate of red blood cell destruction. Liver function may be associated with hemolytic anemia rather than aplastic anemia.

A newly licensed nurse is caring for a client who is at risk for developing diabetes insipidus. Which of the following should be included in the client's plan of care? Measure blood glucose levels every 4 hr Monitor for oliguria Initiate fluid restrictions Check urine specific gravity

4. Check urine specific gravity The urine will become dilute, resulting in a low specific gravity in a client with diabetes insipidus; therefore, the nurse should check the urine specific gravity.

A nurse is assessing a client who will undergo abdominal surgery in 2 hr. The nurse finds that the client has mild anxiety about the surgery, last had food and fluids at 2330 the previous evening, and signed the surgical consent 2 days ago. Which of the following is an appropriate nursing action regarding these findings? Call the anesthesiologist to sedate the client. Notify the surgeon of the client's food and fluid consumption. Witness the surgical consent. Document the findings in the client's medical record.

4. Document the findings in the client's medical record. Whenever a nurse collects data from a client, documentation is essential. However, in this case, all these findings are expectations for a client who is preoperative, so there is no need for the nurse to take any action other than documenting. Mild preoperative anxiety is an expected finding. There is no need to sedate the client at this time. As long as the client has not ingested food or fluids after midnight, there is no need to notify the surgeon. The nurse may only witness a client's signature when the nurse is present at the signing. If there was no witness present, the nurse should notify the surgeon, who will then have to obtain a witnessed consent.

A nurse working for a home health agency is assessing an older adult male client. Which of the following findings is the priority for the nurse to address? 1. Swollen gums 2. Pruritis 3. Urinary hesitancy 4. Dysphagia

4. Dysphagia Dysphagia poses the greatest safety risk to the client because it can cause choking, or result in aspiration of food or liquids leading to pneumonia and respiratory compromise. This is the priority finding for the nurse to address.

A nurse is caring for a client who has just been diagnosed with cancer of the colon. The client asks the nurse several questions about what the provider might be planning to do. Which of the following nursing responses is appropriate? Provide the client with articles from the Internet that explain colon cancer. Assure the client that the provider will explain what has been planned. Tell the client the various options available for treatment. Encourage the client to write down questions to ask the provider.

4. Encourage the client to write down questions to ask the provider. The nurse does not know the answers to the client's questions, so helping the client to prepare questions for the provider addresses the client's needs. It is not within the nurse's scope of practice to discuss treatment options with the client. Assuring the client that the provider will explain what has been planned blocks communication by putting the client's concern on hold and giving false reassurance.

A hospice nurse makes weekly visits to a client who has terminal cancer. For several weeks, the client has been taking morphine sulfate for pain relief. At today's visit, the client reports to the nurse that the usual dose of morphine does not seem to be working. The nurse should understand that the most likely explanation is that the client has... Tolerance of pain is the same in all people. Tolerance of pain is determined by a person's genetic makeup. Pain perception is the same in all people. Expression and perception of pain vary widely from person to person.

4. Expression and perception of pain vary widely from person to person. Pain perception is an individual experience. Research indicates that pain tolerance and perception vary widely among individuals, even within cultures.

A nurse is teaching an older adult client who has diabetes mellitus about preventing the long-term complications of retinopathy and nephropathy. Which of the following instructions should the nurse include? Have an eye examination once per year. Examine your feet carefully every day. Wear compression stockings daily. Maintain stable blood glucose levels.

4. Maintain stable blood glucose levels. Keeping blood glucose under control is the client's best protection against long-term complications of diabetes, as increased blood sugar contributes to neuropathic disease, microvascular complications, and risk factors for macrovascular complications. Having an eye exam is an important practice that helps with early detection of complications. It does not prevent them, however. Examining the feet is an important measure for a client who has neuropathy to use to detect skin alterations, but it does not prevent neuropathy. Compression stockings can impair circulation and increase the risk for complications. Clients who have diabetes should not wear them routinely.

A family member is instructed by a nurse on the interventions for safe swallowing for a client who has residual effects from a stroke. Which of the following concepts are most important for the family members to understand? Offer mouth care before meals Place food in the unaffected side of the mouth. Encourage the client to take small bites and chew food thoroughly. Place the client in the upright position to facilitate swallowing.

4. Place the client in the upright position to facilitate swallowing. The greatest risk to this client is injury from aspiration; therefore, the most important concept for the family members to implement is to place the client in the upright position for meals to facilitate swallowing and prevent aspiration. All other options are correct instructions, but not the most important concept.

A nurse is caring for a client who has pneumonia. The client's oxygen saturation is 85%. Which of the following should the nurse do first Administer oxygen at 2 L/min. Notify the provider. Encourage coughing and deep breathing. Raise the head of the bed.

4. Raise the head of the bed. Elevating the head of the bed can reduce the client's workload and minimize fatigue. Raising the head of the bed uses gravity to drop the abdominal organs away from the diaphragm, which provides increased expansion of the lungs. The client should be assessed further and less invasive interventions implemented before applying oxygen at 2 L/min. The nurse may notify the provider if the oxygen saturation does not improve with some interventions first. Coughing and deep breathing prevents respiratory infection, but is not effective in immediate increasing of oxygen saturation.

A nurse is caring for a preoperative client who is sedated and awaiting surgery. While reviewing the client's preoperative forms, the nurse notes that the consent form has been signed by the client but has not been witnessed. The nurse should... proceed with client preparation because the signed form is valid without a witness. ask the client to resign the form so that the nurse can sign as the witness. sign as a witness after verifying the client's signature with the client. notify the nurse manager and the provider.

4. notify the nurse manager and the provider. The surgery cannot be performed without a signed, witnessed consent form. The nurse manager and the provider should be informed of the situation. Unless the original witness can be found and her signature obtained, a decision may be made to delay the surgery until the sedation wears off and the client can legally sign in the presence of a witness.

A patient with a genetic mutation of BRCA1 and a family history of breast cancer is admitted to the surgical unit where she is scheduled for that day for a bilateral simple mastectomy. What is the reason for this procedure? A. prevent breast cancer B.Diagnose breast cancer C. Cure or control breast cancer D. provide palliative care for untreated breast cancer

A

A patient with bilateral cataracts is schedulded for an extracapsular cataract extraction with an intraocular lens implantation of one eye. What should be done by the nurse preoperatively? A. assess the visual acuity in the unoperated eye to plan the need for postoperative assistance B. inform the patient that the operative eye will need to be patched for 3-4 days postoperatively C. assure the patient that vision in the operative eye will be improved to near normal on the first postoperative day D. teach the patient routine coughing and deep-breathing techniques to prevent respiratory complications

A

A patient with ulcerative colitis has a total proctocolectomy with formation of a terminal ileum stoma. What is the most important nursing intervention for the patient postop? A. Measure the ileostomy output to determine fluid balance B. Change the ileostomy appliance every 3-4 hours to prevent leakage onto skin C. emphasize that the ostomy is temporary and the ileum will be reconnected when the large bowel heals D. teach the patient about the high-fiber, low-carbohydrate diet required to maintain normal ileostomy drainage

A

A pt with a gastric outlet obstruction has been treated with NG decompression. After the first 24 hours, pt develops nausea and increased upper abbominal bowel sounds. What is the priority action of the nurse? A. check the patency of the NG tube B. place the patient in the recumbent position C. check the vital signs D. encourage te patient to deep breathe and consciously relax

A

A pt with a positive breast biopsy tells the nurse that she read about Tamoxifen online and asks about its use. The best response by the nurse is A. treatment of choice if the tumor has receptors for estrogen on its cells B. primary treatment for breast cancer if axillary lymph nodes are + for cancer C. used only to prevent the development of new tumors in women with a high risk of cancer D. Has not been shown to increase the risk for uterine cancer and is only used when other treatment has not worked

A

During a preoperative physical exam, the nurse is alerted to the possibility of compromised respiratory function during or after surgery in a patient with which problem? A. obesity B. dehydation C. enlarged liver D. decreased peripheral pulses

A

During the acute phase of a stroke, the nurse assesses the patient's vital signs and neurologic status every 4 hours. A cardiovascular sign that the nurse would see as the body attempts to increase cerebral blood flow is a. hypertension b. fluid overload c. cardiac dysrhythmias d. S3 and S4 heart sounds

A

Four days following a stroke, a patient is to start oral fluids and feedings. Before feeding the patient, the nurse should first a. check the patient's gag reflex b. order a soft diet for the patient c. raise the head of the bed to sitting position d. evaluate the patient's ability to swallow small sips of ice water

A

Regardless of precipitating factor, the injury to mucosal cells in PUD is caused by: a. acid back-diffusion into the mucosa b. ammonia formation in the mucus wall c. breakdown of gastric mucosal barrier d. release of histamine for cells

A

The best prognosis is indicated in the patient with breast cancer when diagnosis reveals A. negative axillary lymph nodes B. aneuploid DNA tumor content C. cells with high S phase fractions D an ER - and PR - tumor

A

The nurse determines that teaching about pernicious anemia has been effective when the patient says, A. this condition can kill me unless I take injections of the vitamin for the rest of my life B. my symptoms can be completely reversed after I take vit b12 supplements C. if my anemia does not respond to cobalamin therapy, my only other alternative is a bone marrow transplant D. the least expensive and most convenient treatment of pernicious anemia is to use diet with foods high in cobalamin

A

The nurse recognizes that thrombolytic therapy for the treatment of an MI has not been successful when the patient displays which manifestation? A. continues to have chest pain B. has an increased in CK-MB levels within 3 hours of therapy C. develops GI or GU bleeding D. Develops premature ventricular contractions and ventricular tachycardia during treatment

A

The nurse suspects stable angina rather than MI pain in the patient who reports that his chest pain A. is relieved by nitroglycerin B. Is a sensation of tightness or squeezing C. does not radiate to his neck, back, or arms D. is precipitated by physical or emotional exertion

A

What is the most important method to identify the presence of infection in a neutropenic patient? A. frequent temp monitoring B. routine blood and sputum cultures C. assessing for redness/swelling D. monitoring WBC count

A

Which word identifies a mutation of protooncogenes? A. oncogenes B. retrogenes C. oncofetal antigens D. tumor necrosis factors

A

a newly diagnosed patient diagnosed with right-sided brain stroke has a nursing diagnosis of disturbed visual sensory perception related to homonymous hemianopsia. Early in the care of the patient, what should the nurse do? A. place objects on the right side within the patient's field of vision B. approach the patient from the left side to encourage the patient to turn the head C. place objects on the patient's left side to assess the patient's ability to compensate D. patch the affected side to encoruage the patient to turn the head to scan the environment

A

a patient is admitted to the hospital with a diagnosis of Cushing's. On physical assessment of the patient, what should the nurse expect to find? A. hypertension/peripheral edema/petechiae B. weight loss/buffalo hump/moon face w acne C. Abdominal and buttock straie/truncal obesity/hypotension D. anorexia/signs of dehydration/hyperpigmentation of skin

A

during an annual health assessment of a 66 year old patient at the clinic, the patient tells the nurse he has not had the pneumonia vaccine. What should the nurse advise him about the best way for him to prevent pneumonia? A. obtain a vaccine now and get a booster 12 months later B. seek medical care and antibiotic therapy for all upper respiratory infections C. obtain the pneumococcal vaccine if he is exposed to individuals with pneumonia D. obtain only the influenza vaccine every year because he should have immunity to the pneumococcus because of his age

A

what is the most likely reason that a BUN would be increased in a patient? A. impaired renal function B. has not eaten enough protein C. has decreased urea in urine D. may have nonrenal tissue hypertrophy

A

what type of stroke is associated with endocardial disorders, has a rapid onset, and is likely to occur during activity? A. embolic B. thrombotic C. intracerebral D. subrachnoid

A

when obtaining a health history from a 76 year old patient with suspected CAP, what does the nurse expect the patient or caregiver to report? A. confusion B. a recent loss of consciousness C. an abrupt onset of fever and chills D. a gradual onset of headache and sore throat

A

when obtaining a nursing history from the patient with colorectal cancer, the nurse should specifically ask the patient about A. dietary intake B. sports involvement C. environmental exposure to carcinogens D. long term use of NSAIDs

A

what are characteristics of vasospastic disease (Raynaud's) (select all that apply) A. predominant in young females B. May be associated with autoimmune disorders C. Precipitated by exposure to cold, caffeine, tobacco D. involves small cutaneous arteries of the fingers E. inflammation of small and medium-sized arteries and veins F. episodes involve white, blue, and red color changes of fingertips

A, B, C, D, F

Cancer cells go through stages of development. What accurately describes the stage of promotion (select all that apply) A. obesity is a promoting factor B. the stage is characterized by increased growth rate and metastasis C. withdrawal of promoting factors will reduce the risk of cancer development. D. Tobacco smoke is a complete carcinogen is that is capable of both initiation and promotion.

A, C, D

Which statements describe thrombocytopenia (select all that apply) A. patients with platelet def can have internal/external hemorrhage B. the most common acquired is TTP C. ITP is characterized by platelet destruction from the spleen D. TTP is characterized by decreased platelets and RBCS E. Classic assessment manifestation is ecchymosis

A, C, D,

Which nursing actions are completed by the scrub nurse? (select all that apply) A. prepares instrument table B. documents intraoperative care C. remains in the sterile area of the OR D. checks mechanical and electrical equipment E. passess instruments F. monitors blood/fluid loss.urine output

A, C, E

The patient with diabetes has a blood glucose of 248. which manifestations in the patient would the nurse understand as being related to this blood glucose level? A. headache B. unsteady gait C. abdominal cramps D. emotional changes E. increase in urination F. weakness and fatigue

A, C, E, F

Which characteristics of glaucoma are associated with only acute primary angle-closure glaucoma? (select all that apply) A. caused by lens blocking papillary opening B. treated with trabeculoplasty C. causes loss of central vision with corneal edema D. treated with beta blockers e. causes sudden severe eye pain and N/V F. treated with hyperosmotic oral and IV fluids to lower IOP

A, C,E, F

A postmenopausal female patient who smokes and takes hormone therapy just had a hip replacement and is going back to the clinical unit. Which factors presented in this patient increases her risk for developing VTE related to Virchow's triad? A. smoking B. IV therapy C. dehydration D. estrogen therapy E. orthopedic surgery F. prolonged immobilization

A, D, E, F

the nurse finds a patient in bed having a tonic-clonic seizure. During the seizure activity, what should the nurse do FIRST (select all that apply) A. loosen restrictive clothing B. turn the patient on their side C. protect the patient's head from injury D. place a padded tongue blade in between the patient's teeth E. restrain the patient's extermities

A,B,C

For the patient hospitalized with inflammatory bowel disease (IBD), which treatments would be used to rest the bowel (select all that apply)? a. NPO d. Sedatives b. IV fluids e. Nasogastric suction c. Bed rest f. Parenteral nutrition

A,B,E,F

how do microorganisms reach the lungs and cause pneumonia? (select all that apply) A. aspiration B. lymphatic spread C. inhalation of microbes in air D. touch contact with the infectious microbes E. hematogenous spread from infections elsewhere in the body

A,C,E

A 20 year old patient with Crohn's disease comes to the clinic with persistent diarrhea. What are common characteristics of Crohn's? (select all that apply) A. weight loss B. rectal bleeding C. abdominal pain D. toxic megacolon E. has segmented distribution F. involves the entire thickness of the bowel wall

A,C,E,F

What characteristics should the nurse expect in a patient with sensorineural loss? (select all that apply) A. head trauma. B. related to otitis media C. exposed to noise trauma D. linked with otosclerosis E. patient's mother has hearing loss F. associated with meniere's

A,C,E,F

when teaching the patient about the new diagnosis, which characteristics are only related to primary open angle glaucoma? (select all that apply) A. gradual loss of peripheral vision B. treated with iridectomy C. causes loss of central vision with corneal edema D. may be caused by increased production of aqueous humor E. treated with cholinergic agents F.reisistance to aqueous outflow through trabecular meshwork

A,D,F

A patient is scheduled for a surgery at an ambulatory surgery center. An advantage of performing surgery here is a decreased need for A. diagnostic studies and perioperative medications B. preoperative and postoperative teaching by the nurse C. psychologic support to alleviate fears of pain and discomfort D. preoperative nursing assessment related to possible risks and complications

A.

A patient is scheduled for hip replacement surgery in the early afternoon and is NPO. He ingests a breakfast tray with clear liquids on the morning of surgery. What response does the nurse expect when the ACP is notified? A. surgery will be done as schedulded B. surgery will be rescheduled for tomorrow C. surgery will be postponed 8 horus after fluid intake D. An NG tube will be inserted to remove the fluids

A.

During the nursing assessment of a patient with anemia, what specific information should the nurse ask the patient about? A. stomach surgery B. recurring infections C. corticosteroid therapy D. oral contraceptive use

A.

The PACU nurse applies warm blankets to a post-op patient who is shivering and has a body temperature of 96 degrees Fahrenheit. What treatment also may be used to treat the patient? a. oxygen b. vasodilating drugs c. antidysrhythmic drugs d. analgesics or sedatives

A.

The health care provider prescribes spironolactone for the patient with heart failure. What diet modifications related to the use of this drug should the nurse include in the patient teaching? A. decrease both sodium and potassium intake B. increase calcium intake and decrease sodium intake C. decrease sodium intake and increase potassium intake D. decrease sodium intake by using salt substitutes for seasoning

A.

The nurse asks a preop patient to sign a consent form as specified by the surgeon and then signs the form after the pt does so. By this action, what is the nurse doing? A. witnessing the patient's signature B. obtaining informed consent from the patient for the surgery C. verifying that the consent for surgery is voluntary and informed D. ensuring that the patient is mentally competent to sign

A.

The nurse is presenting a community education program related to cancer prevention. Based on current cancer death rates, the nurse emphasizes what as the most important preventative action for both men and women? A. smoking cessation B. routine colonoscopies C. frequent imaging tests D. regular examination of reproductive organs

A.

The patient has chronic pain that is no longer relieved with oral morphine. Which medication would the nurse expect to be ordered to provide better pain relief for this patient? A. Fentanyl B. Hydrocodone C. Intranasal butorphanol D. Morphine sustained-release

A.

To prevent hyperglycemia or hypoglycemia with exercise, the nurse teaches the patient using glucose-lowering agents about the best time of exercise. A. Plan activity and food intake related to blood glucose levels B. when blood glucose is great than 250 C. when glucose monitoring reveals that the blood glucose is in the normal range D. when blood glucose levels are high, because exercise always has a hypoglyemic effect

A.

What early manifestations is the patient with primary hypertension likely to report? A. no symptoms B. cardiac palpitations C. dyspnea on exertion D. dizziness and vertigo

A.

What is a primary reason that the normal immune response fails to contain the HIV infection? A. CD4T cells become infected with HIV and are destroyed B. B cells are inactivated which prevents the production of HIV antibodies C. NAtural killer cells are destroyed by the virus before the immune system can be activated D. monocytes ingest infected cells, differentiate into macrophages, and shed viruses in body tissues

A.

What is the PRIMARY goal of the circulating nurse during prep of the OR room, transferring/positioning the patient, and assisting the anesthesia team? A. avoiding any type of injury to the patient B. maintaing a clean environment for the patient C. providing for patient comfort and a sense of well-being D. preventing breaks in aseptic technique by the sterile members of the team

A.

What is the primary advantage of the use of midazolam as an adjunct to general anesthesia? A. amnestic effect B. analgesic effect C. prolonged action D. antiemetic effect

A.

when the patient who is diagnosed with an MI si not relieved of chest pain with IV NTG, which medication will the nurse expect to be used? A. IV morphine sulfate B. Calcium channel blockers C. IV administration of amiodarone D. ACE inhibitors

A.

which statement accurately describes Grave's disease? A. exophthalamos occurs in Grave's B. it is an uncommon form of hyperthyroidism C. manifestations of hyperthyroidism include tissue sensitization to the SNS D. diagnostic testing in the patient will indicate an increase in TSH

A.

A nurse is caring for a client diagnosed with diabetes mellitus and observes mild hand tremors. Which of the following actions should the nurse take after obtaining a glucometer reading of 60 mg/dL? Administer 10-15 g of carbohydrates. Retest the blood glucose. Administer 1 mg of glucagon (GlucaGen) IM. Call for emergency assistance.

Administer 10-15 g of carbohydrates. The first step in preventing the blood glucose from dropping further in a client who has diabetes mellitus would be to administer 10-15 g of carbohydrates. The following will provide 10-15 g of carbohydrates: glucose tablets/gel (dosage provided on label), 4 oz orange juice, 8 oz skim milk, 6 saltine crackers, 3 graham crackers, or 6 to 10 hard candies. If the client experiences a decline in mental status, IM or IV glucagon (GlucaGen) can be given. Retesting blood glucose will delay treatment. The client is symptomatic, and the glucose level is below the expected reference range. Administering 10-15 g of carbohydrates will increase the blood glucose and decrease the shakiness associated with hypoglycemia. Although a low blood glucose can be serious, this client is not experiencing symptoms consistent with the need for emergency help. The nurse can act promptly to increase the blood glucose level of the client to prevent hypoglycemic shock.

A female patient with a UTI also has renal calculi. The nurse knows that these are most likely which kind of stone? A.cystine B. struvite C. uric acid D. calcium phosphoate

B

A patient with ulcerative colitis undergoes the first phase of a total proctocolectomy with ileal pouch and anal anastomosis. On postoperative assessment of the patient, what should the nurse expect to find? a. A rectal tube set to low continuous suction b. A loop ileostomy with a plastic rod to hold it in place c. A colostomy stoma with an NG tube in place to provide pouch irrigations d. A permanent ileostomy stoma in the right lower quadrant of the abdomen

B

A small lesion is found in the patient's lung when an x-ray was performed for cervical spine pain. What is the definitie method for determining if this lesion is malignant? A. lung scan B. tissue biopsy C. oncofetal antigens in the blood D. CT or PET scan

B

Mitoxantrone is being considered as treatment for a patient with progressive-relapsing MS. The nurse explains that a disadvantage of this drug compared with other drugs used for MS is what? A. it must be given sucbutaneously everyday B. it has a lifetime dose limit because of cardiac toxicity C> it is a muscle relaxant that increases the risk for drowsiness D. it is an anticholinergic that causes urinary incontinence

B

Most organ damage in hypertension is related to what? A. increased fluid pressure exerted against organ tissue B. artherosclerotic changes in vessels that supply the organs C. erosion and thinning of blood vessels in organs from constant pressure D. increased hydrostatic pressure causing leakage of plasma into organ interstitial spaces

B

Nursing interventions for the patient with aplastic anemia are directed toward the prevention of which complications A. fatigue/dyspnea B. hemorrhage/infection C. thromboemboli/gangrene D. cardiac dysrhythmias./HF

B

On examining a patient 8 hours after having surgery to create a colostomy, what should the nurse expect to find? a. Hyperactive, high-pitched bowel sounds b. A brick-red, puffy stoma that oozes blood c. A purplish stoma, shiny and moist with mucus d. A small amount of liquid fecal drainage from the stoma

B

The nurse is counseling a group of individuals over the age of 50 with average risk for cancer about screening for cancer. Which screening recommendation should be performed to screen for colorectal cancer? A. barium enema every year B. colonscopy every 10 years C. fecal occult blood every 5 years D. annual PSA and digital rectal exam

B

The patient has HTN and just experienced an MI. What type of medication should the nurse expect to be added to decrease the workload on his heart? A. ACE inhibitor B. Beta blocker C. calcium channel blocker D. ARB

B

The patient is diagnosed with SIADH. What manifestations should the nurse expect to find? A. decreased body weight B. decreased urinary output C. Increased plasma osmolarity D. Increased serum sodium levels

B

What is the rationale for using preop checklists on the day of surgery? A. patient is correctly identified and preop medications have been administered B. All preop orders and procedures have been carried out and documented C. voiding is the last procedure before the patient is transported to the operating room D. patient's families have been informed as to where they can wait for the patient

B

Myocardial ischemia occurs as a result of increased oxygen demand and a decreased oxygen supply. What factors/disorders result in an increased oxygen demand? (select all that apply) A. hypovolemia or anemia B. increased cardiac workload with aortic stenosis C. narrowed coronary arteries D. angina in the patient with atheroscleroric coronary arteries E. Left ventricular hypertrophy caused by chronic hypertension F. SNS stimulation by drugs, emotions, exertion

B, C, D, E, F

a patient has been diagnosed with hypoparathyroidism. What manifestations should the nurse expect to observe? (select all that apply) A. skeletal pain B. dry, scaly skin C. personality changes D. abdominal cramping E. cardiac dysrhythmias F. muscle spasms adn stiffness

B, C, D, E, F

What are characteristics of peripheral artery disease? (select all that apply) A. Pruritus B. thickened, brittle nails C. dull ache in calf or thigh D. decreased peripheral pulses E. pallor on elevation of the legs F. ulcers over bony prominences on toes and feet

B, D, E, F

A patient with PAD has ineffective peripheral tissue perfusion. What should the nurse include in the teaching plan for this condition? (select all that apply) A. apply cold compresses when legs become swollen B. wear protective footwear and avoid hot/cold extremes C. Walk at least 30 minutes per day, at least 3x per week D. use nicotine replacement therapy as a substitute for smoking E. Inspect lower extremities for pulses, temperature, and any injury

B,C, E

A patient with SIADH is treated with water restriction and administration of IV fluids. The nurses evaluates that treatment has been effective when the patient experiences a. increased urine output, decreased serum sodium, and increased urine specific gravity b. increased urine output, increased serum sodium, and decreased urine specific gravity c. decreased urine output, increased serum sodium, and decreased urine specific gravity d. decreased urine output, decreased serum sodium, and increased urine specific gravity

B.

A patient with multiple injuries resulting from an automobile accident tells the nurse he has "bad" pain but that he can "tough it out" and does not require pain medication. To gain the patient's participation in pain management, what is most important for the nurse to explain to the patient? A. patients have a responsiblity to keep the nurse informed about pain B. unrelieved pain has many harmful effects on the body that can impair recovery C. using pain medications rarely leads to addiction when they are used for actual pain. D. nonpharmacologic therapies can be used to relieve his pain if he is afraid to use pain medications

B.

What accurately describes the pathophysiology of CAD? A. partial or total occlusion of the coronary artery occurs during the final stage of raised fibrous plaque. B. endothelial alteration may be caused by chemical irritants such as hyperlipidemia or by tobacco use. C. collateral circulation in the coronary circulation is more likely to present in the young patient with CAD D. the leading theory of atherogenesis proposes that infection and fatty dietary intake are the basic underlying causes of atherosclerosis

B.

What is the physical environment of a surgical suite primarily designed to promote? A. electrical safety. B. medical asepsis C. comfort of the patient D. communciation among the surgical team

B.

When a patient is undergoing brachytherapy, what is importatn for the nurse to be aware of when caring for this patient? A. patient will undergo simulation to identify and mark the field of treatment B. the patient is a source of radiation, and personnel must wear film badges during care C. the goal of this treatment is only palliative, and the patient should be aware of the expected outcome D. computerized dosimetry is used to determine the maximum dose of radiation to the tumor within an acceptable dose to normal tissue

B.

When teaching the patient with PAD about modifying risk factors associated with this condition, what should the nurse emphasize? A. amputation is the ultimate outcome if patient does not alter lifestyle behaviors B. modifications will reduce the risk of atherosclerotic conditions such as stroke C. risk-reducing behaviors inititated after angioplasty can stop the progression of the disease D. maintenance of normal body weight is the most important factor in controlling the disease

B.

Which patient is ready for discharge from Phase I PACU care to the clinical unit? A. arouses easily, pulse is 112 bpm, resp rate is 24, dressing is saturated, SpO2 88% B. awake, vital signs stable, dressing is dry/intact, no respiratory depression, SpO2 is 92% C. difficult to arouse, pulse is 52, resp rate 22, dressing is dry/ontact, SpO2 is 91% D. arouses, BP higher than preoperatively, resp rate is 10, no excess bleeding, SpO2 90%

B.

to prevent agitation during the patient's recovery from anesthesia, when should the nurse begin orientation explanations? a. when the patient is awake b. when the patient first arrives in the PACU c. when the patient becomes agitated or frightened d. when the patient can be aroused and recognizes where he or she is

B.

what condition should the nurse anticipate that may occur during epidural and spinal anesthesia? A. spinal headache B. hypotension and bradycardia C. loss of consciousness and seizures D. downward extension of nerve block

B.

what is the pathophysiologic mechanism that results in the pulmonary edema of left-sided heart failure? A. increased right ventricular preload B. Increased pulmonary hydrostatic pressure C. Impaired alveolar oxygen and carbon dioxide exchange D. Increased lymphatic flow of pulmonary extravascular fluid

B.

when transporting an inpatient to the surgical department, a nurse from another area of the hospital is able to access which area? A. sterile core B. holding area C. corridors of surgical suite D. unprepared operating room

B.

which serum lipid elevation, along with elevated LDL, is strongly associated with CAD? A. apolipoproteins B. fasting triglycerides C. total serum cholesterol D. HDL

B.

During the immediate post op period following a modified radical mastectomy, the nurse initially institutes which exercises for the affected arm? A. have the patient comb her hair with the affected arm B. perform full ROM on the affected arm C. Ask the patient to flex/extend the fingers and wrist of the operative side D. Have the patient crawl her fingers up the wall, raising her arm above her head

C

How should the nurse teach the patient with GERD to control symptoms? A. drink 10-12 ounces of water with each meal B, space 6 small meals a day between breakfast-bedtime C. sleep with the head of the bed elevated on 4 to 6 inch blocks D. perform daily activities of toe-touching and sit ups

C

In a patient with diabetes inspidus, the administration of ADH during a water deprivation test will result in what? A. Decrease in body weight B. increase in urinary output C. decrease in blood pressure D. increase in urine osmolarity

C

The patient is receiving the following medications. Which one is prescribed to relieve symptoms rather than treat a disease? A. corticosteroids B. 6-mercaptopurine C. Antidiarrheal agents D. sulfasalazine

C

What does the nurse include when planning for postural drainage for the patient with COPD? A. schedule the procedure 1 hour before and after meals B. have the patient cough before positioning to clear the lungs C. assess the patient's tolerance for dependent (head down) positions D. ensure that percussion and vibration are performed before positioning the patient

C

What does the nurse include when teaching a patient with newly diagnosed peptic ulcer disease? A. maintain a bland , low-residue diet B. use alcohol and caffeine in moderation and always with food C. eat as normally as possible, eliminating foods that cause pain or discomfort D. avoid milk because it stimulates gastric acid production

C

What is a cause of primary hypothyroidism in adults? A. malignant or benign thyroid nodules B. surgical removal or failure of the pituitary gland C. autoimmune-induced atrophy of the thyroid gland D. surgical removal or radiation of the thyroid gland

C

What is an appropriate food for a stroke patient with mild dysphagia? A. fruit juice B> pureed meat C. scrambled eggs D. fortified milkshakes

C

What should the nurse encourage the patient with diverticulosis to do? A. use antibiotics routinely to prevent future inflammation B. have an annual colonscopy to detect malignant changes in the lesions C. maintain a high-fiber diet and encourage fluid intake of at least 2 L per day D. exclude whole grain breads and cereals from the diet to prevent irritating the bowel

C

What should the nurse include when teaching the patient with COPD about the need for physical exercise? A. all patients with COPD should increase walking gradually up to 20 minutes per day B. a bronchodilator inhaler should be used to relieve exercise-induced dyspnea immediately after exercise C. SOB is expected during exercise but should return to baseline within 5 minutes afterwards D. Monitoring the heart rate before and after exercise is the best way to determine how much exercise can be tolerated

C

When teaching the patient with mild asthma about the use of the peak flow meter, what should the nurse instruct the patient to do? A. carry the flowmeter with the patient at all times in case an asthma attack occurs B. use the flowmeter to check the status of all patient's asthma every time the patient takes quick relief medication C. follow the written asthma action plan if the expiratory flow rate is in the yellow zone D. use the flowmeter by emptying the lungs, closing the mouth around the mouthpiece, and inhaling through the meter as quickly as possible

C

Which intervention should the nurse delegate to the LPN when caring for a patient following an acute stroke? A. assess the neurologic status B. assess the patient's gag reflex C. administer ordered antiHTNsives and platelet inhibitors D. teach the patient's caregivers strategies to minimize unilateral neglect

C

a patient is admitted to the hospital for evaluation and treatment of thrombocytopenia. Which action is most important for the nurse to implement? A. taking temp every 4 hours to assess for fever B. maintaining the patient on strict bed rest to prevent injury C. monitoring the patient for headaches, vertigo or confusion D. removing the oral crusting and scabs with a soft brush 4x a day

C

what is an important health promotion nursing intervention for a middle-aged adult related to glaucoma? A. teach individuals at risk for glaucoma about early signs and symptoms of the disease B. prepare patients with glaucoma for lifestyle changes necessary to adapt to eventual blindness C. promote measurements of intraocular pressure every 2-4 years for early detection and treatment of glaucoma. D. inform patients that glaucoma is curable is eye medications are administered before visual impairment occurs

C

which observation of the patient made by the nurse is the most indicative of Parkinson's disease? A. large, embellished handwriting B. weakness of one leg resulting in a limping walk C. difficulty rising from a chair and beginning to walk D. onset of muscle spasms occurring with voluntary movement

C

which diagnostic test is most useful in differentiating dyspnea related to pulmonary effects of heart failure from dyspnea related to pulmonary disease? A. exercise testing B. cardiac catheterization C. BNP levels D. BUN

C BNP is useful for diagnosing Heart failure

The patient diagnosed with secondary hypertension asks why it is called secondary and not primary. What is the best explanation for the nurse to provide? A. has a more gradual onset than primary hypertension B. does not cause the target organ damage that occurs with primary hypertension. C. Has a specific cause, such as renal disease, that often can be treated by medicine or surgery D. Is caused by age-related changes in BP regulatory mechanisms in people over 65

C,

What are manifestations of acute coronary sundrome? (select all that apply) A. dysrhythmia B. stable agina C. unstable angina D. STEMI E. NSTEMI

C, D, E

Which descriptions are characteristic of iron-def anemia? (select all that apply) A. lack of intrinsic factor B. autoimmune disease C. most common type of anemia D. associated with chronic blood loss E. may occur with removal of the stomach F. may occur with removal of the duodenum

C,D,E,F

the patient is diagnosed with complex focal seizures. Which characteristics are related to complex focal seizures (select all tht apply) A. formerly known as grand mal B. accompanied by incontinence and cheek biting C. psychomotor seizures with repetitive behaviors and lip smacking D. altered memeroy, sexual sensations, and distortions of visual or auditory sensations E. loss of consciousness and stiffening of the body with subsequent jerking of extremities F. often involves behavioral, emotional, and cognitive functions with altered consciousness

C,D,F

A 62 year old woman has prehypertension (138/88) and smokes a pack of cigarettes a day. She has no symptoms of CAD, but has an LDl level of 154. Based on these findings, the nurse would expect that which treatment plan would be used first for this patient? A. Diet and drug therapy B. exercise instruction only C. Diet therapy and smoking cessation D. Drug therapy and smoking cessation

C.

A patient who had major surgery is experiencing emotional stress as well as physiologic stress from the effects of surgery. What can this stress cause? A. diuresis B. hyperkalemia C. fluid retention D. impaired blood coagulation

C.

An ECG performed on a client 4 hours after the onset of chest pain reveals ST segment elevation. What does the nurse recognize that this finding indicates? A. ischemia typical of unstable angina B. lack of permanent damage C. MI associated with prolonged and complete coronary thrombosis D. MI associated with transient or incomplete coronary artery occlusion

C.

An allogenic hematopoietic stem cell transplant is considered as treatment for a patient with leukimia. What information should the nurse include when teaching the patient about this procedure? A. there is no risk for graft-versus-host disease because the donated marrow is treated to remove cancer cells B. bone marrow will be removed, treated, stored, and then reinfused after chemo C. peripheral stem cells are obtained from a donor who has a leukocyte antigen match with the patient D. there is no need for posttransplant proctective isolation because the stem cells are infused directly into the blood

C.

The nurse determines that the treatment of heart failure has been successful when the patient experiences A. weight loss and diuresis B. warm skin and less fatgiue C. clear lung sounds and decreased HR D. absence of chest pain and improved LOC

C.

The nurse monitors the patient receiving treatment for acute decompensated heart failure with the knowledge that marked hypotension is most likely to occur with the IV administration of which medication? A. milrinone B. furosemide C. nitroglycerin D. nitroprusside

C.

To detect and treat the most common complication of MI, the nurse? a. measures hourly urine output. b. auscultates the chest for crackles. c. uses continuous cardiac monitoring. d. takes vital signs q2hr for the first 8 hours

C.

What describes a primary use of immunotherapy in cancer treatment? A. protects normal, rapidly reproducing cells of the gastrointestinal system from damage during chemotherapy B. prevents the fatigue associated with chemotherapy and high-dose radiation C. enhances the effects of the host's immune responses to tumor cells that produce flu-like symptoms D. depresses the immune system and circulating lymphocytes and increases sense of well-being

C.

When the nurse asks a preop patient about her allergies, the patient reports a history of allergies to fruits. What should the nurse do next? A. note this information in the patient's record as food allergies B. Place an allergy alert wristband that identifies the specific allergy on the patient C. Ask the patient to describe the nature and severity of any allergic responses from these agents D. Notift the anesthesia provider because the patient may be allergic to the anesthesia agent

C.

in the patient with chest pain, which results differentiate unstable angina from an MI? A. ECG changes present at the onset of pain B. chest x-ray indicating left ventricular hypertrophy C. serum troponin levels increased 4-6 hours after the onset D. Creatinine kinase MB elevations that peak 6 hours after the infarct

C.

A 52 year old man is admitted to the emergency department with severe chest pain. On what basis would the nurse expect an MI? A. pale, cool, clammy skin B. Reports nausea and vomited once at home C. he is anxious and has a feeling of impending doom D. he has had no relief or pain with rest or position chagne

D

A 78 year old patient asks the nurse why he has to urinate so much at night. The nurse should explain to the patient that as an older adult, what may contribute to his nocturia? A. decreased renal mass B. decreased detrusor muscle tone C. decreased ability to conserve sodium D. decreased ability to concentrate urine

D

A patient with suprapubic pain and symptoms of urinary frequency and urgency has two negative urine cultures. What is one assessment finding that would indicate internal cystitis? A. residual urine > 200 mL B. large, atonic bladder on urodynamic testing C. voiding pattern that indicates psychogenic urinary retenion D. Pain with bladder filling that is relieved by urination1

D

A patient's wife asks the nurse why her husband did not receive the clot busting medication (TPA) she has been reading about. Her husband is diagnosed with a hemorrhagic stroke. What is the best response? A. he didn't arrive within the timeframe for that therapy B. not everyone is eligible for this drug. has he had surgery recently? C. you should discuss the treatment of your husband with his doctor D. the medication you are talking about dissolves clots and could cause more bleeding in your husband's brain

D

Following a procedure, the patient develops dumping syndrome. The nurse should explain that the symptoms associated with this problem are caused by A. distention of the smaller stomach by too much food and fluid intake B. hyperglycemia caused by uncontrolled gastric emptying into the small intestine c. irritation of the stomach lining by reflux of bile salts because the pylorus has been removed D. movement of fluid into the small bowel from concentrated food and fluids moving radpily into the intestine

D

Following the teaching of foot care to a diabetic patient, the nurse determines that additional instruction is needed when the patient says, a. "I should wash my feet daily with soap and warm water." b. "I should always wear shoes to protect my feet from injury." c. "If my feet are cold, I should wear socks instead of using a heating pad." d. "I'll know if I have sores or lesions on my feet because they will be painful."

D

How do generalized seizures differ from focal seizures? A. focal seizures are confined to one side of the brain and remain focal in nature B.generalized seizures result in LOC, whereas focal seizures do not C. generalized seizures result in temporary residual deficits during the postictal phase. D. generalized seizures have bilateral synchronus epileptic discharges affecting the whole brain at onset of the seizure

D

In the patient with heart failure, which medications/treatments require careful monitoring of the patient's serum potassium level to prevent further cardiac dysfunction? (select all that apply) A. enalapril B. furosemide C. nesiritide D. spironolactone E. metoprolol

D

Lispro insulin (Humalog) and NPH (Humulin N) insulin is ordered for a patient with newly diagnosed type 1 diabetes. The nurse knows that when lispro insulin is used, when should it be administered? A. only once a day B. 1 hour before meals C. 30-45 minutes before meals D. at mealtime or within 1 minutes of meals

D

The acronym FACES is used to help teach patients to identify early symptoms of heart failure. What does this acronym mean? A. Frequent Activity Leads to Cough in the Elderly and Swelling B. Factors of Risk: Activity Cough Emotional upsets Salt intake C. Follow activity plan, Continue Exercise, know Signs of problems D. Fatigue, limiation of Activities, chest Congestion/cough, Edema, Sob

D

The health care provider orders a pulmonary angiogram for a patient admitted with dyspnea and hemoptysis. For which problem is this test most commonly used as a diagnostic measure? A. TB B. cancer of the lung C. airway obstruction D. pulmonary embolism

D

While assisting a patient with intermittent asthma to identify specific triggers of asthma, what should the nurse explain? A. food and drug allergies do not manifest in respiratory symptoms B. exercise-induced asthma is seen only in inidividuals with sensitivity to cold air C. asthma attacks are psychogenic in origin and can be controlled with relaxation techniques D. viral upper respiratory infections are a common precipitating factor in acute asthma attacks

D

a patient with Grave's asks the nurse what caused the disorder. What is the best response by the nurse? A. the cause is unknown, although it can be genetic B. it is associated with goiter formation from an iodine deficiency over a long period of time C. Antibodies develop against thyroid tissue amd destroy it, causing a def in thyroid hormone D. in genetically susceptible persons, antibodies are formed that cause excessive thyroid hormone secretions

D

a patient with a new ileostomy needs discharge teaching. what should the nurse plan to include in the teaching? A. pouch can be used up to 2 weeks before changing it B. decrease the amount of fluid intake to decrease the amount of drainage C. Pouch can be removed when bowel movements have been regulated. D. if leakage occurs, promptly remove the pouch, clean the skin, and apply a new pouch

D

tachycardia that is a response of the SNS nervous system to the pain of ischemia is detrimental because it increases oxygen demand and A. increases cardiac output B. causes reflex hypotension C. may lead to atrial dysrhythmias D. impairs perfusion of the coronary arteries

D

the classic manifestations associated with Parkinson's are tremor, rigidity, akinesia, and postural instability. What is a consequence related to rigidity? A. shuffling gait B. impaired handwriting C. lack of postural stability D. muscle soreness and pain

D

the incidence of an ischemic stroke in patients with TIAs and other risk factors is reduced with the administration of what mediation? A. nimodipine B. furosemide C. warfarin D. daily low-dose aspirin

D

what is the name of a tumor from the embryonal mesoderm tissue of origin located in the anatomic site of the meninges that has malignant behavior? A. meningitis B. meningioma C. meningocele D. meningeal sarcoma

D

which patient probably has the highest risk of breast cancer A. 60 year old obese man B. 58 year old woman with sedentary lifestyle C. 55 year old woman with fibrocystic breast changes D. a 65 year old woman with a sister diagnosed with breast cancer

D

A postop 68 year old opioid-naive patient is receiving morphine by PCA for postoperative pain. What is the rationale for not inititating the PCA analgesic with a basal dose as well? A. opioid overdose B.Nausea/vomiting C. lack of pain control D. adverse respiratory outcomes

D.

The patient will be placed under moderate sedation to allow realignment of a fracture in the emergency department. When the family asks about this anesthesia, what should the nurse tell them? A. includes inhalation agents B. induces high levels of seation C. frequently used for traumatic injuries D. patients remain response and breathe without assistance

D.

Tobacco smoke causes defects in multiple areas of the respiratory system. What is a long term effect of smoking? A. bronchospasm and hoarseness B. decreased mucus secretions and coughing C. increased function of alveolar macrophages D. increased risk of infection and hyperplasia of mucous glands

D.

What defect in cellular proliferation is more involved in the development of cancer? A. a rate of cell proliferation that is more rapid than that of normal body cells. B. Shortened phases of cell life cycles with an occassional skipping of G1 or S phases C. Rearrangement of stem cell RNA that causes abnormal cellular protein synthesis D. indiscriminate and continuous proliferation of cells with loss of contact inhibition

D.

What factor differentiates a malignant tumor from a benign tumor? a. it causes death b. it grows at a faster rate c. it is often encapsulated d. it invades and metastasizes

D.

When examining a patient's breasts, the nurse notes multiple lumps. To assess the patient further, what is the most appropriate question by the nurse A. do you have a high caffeine intake? B. when did you last have a mammogram C. is there a history of breast cancer in your mother or sisters D. do the size/tenderness of the lumps change with your menstrual cycle

D.

Which diagnostic test is the most accurate and advantageous in terms of time and expense in diagnosing malignant breast disorders? A. Mammography B. excisional biopsy C. fine-needle aspiration D. core needle biopsy

D.

Which drainage is drained with a Hemovac? A. bile B. urine C. gastric contents D. wound drainage

D.

Which treatment is used first for the patient with a confirmed MI to open the blocked artery within 90 minutes of arrival to the facility? A. TMR B. Stent placement C. coronary artery bypass graft D. percutaneous coronary intervention

D.

A nurse is admitting a client who is having an exacerbation of his asthma. When previewing the provider's orders, the nurse recognizes that clarification is needed for which of the following prescribed medications? Propranolol (Inderal) Theophylline (Theo-Dur) Montelukast (Singulair) Prednisone (Deltasone)

Propranolol (Inderal) Medications that block beta-2 receptors, such as Inderal, are contraindicated in clients with asthma. All other options are medications used in treatment of athsma.

A nurse is teaching a client who has asthma the appropriate use of inhaled beclomethasone (Beclovent). To avoid complications related to the use of beclomethasone, the nurse should encourage the client to... check the pulse after medication administration. take the medication with meals. rinse the mouth after administration. limit caffeine intake.

rinse the mouth after administration. Use of glucocorticoids by metered dose inhaler can allow a fungal overgrowth in the mouth. Rinsing the mouth after administration can lessen the likelihood of this complication. Beclomethasone, a glucocorticoid, does not cause cardiac side effects. Oral, but not inhaled, glucocorticoids should be administered with food. Caffeine does not interact with beclomethasone and is not contraindicated.

A patient comes to the ED immediately after experiencing numbness of the face and an inability to speak but while they wait for examination, the symptoms go away. Why should the nurse emphasize to the patient that treatment is still necessary? A. the pt has probably experienced an asymptomatic lacunar stroke B. the symptoms will likely return and progress to worsening neurologic deficit in the next 24 hours C. neurologic deficits that are transient occur most often as a result of small hemorrhages that clot off D. the patient has experienced a TIA which is a sign of progressive cerebrovascular disease

D

A patient is admitted to the hospital due to a stroke. To determine the size and location and to ascertain whether a stroke is ischemic or hemorrhagic, what will the nurse anticipate that the HCP will request? A. lumbar puncture B. cerebral angiography C. MRI D. CT scan

C

What noninvasive diagnostic test can be used to differentiate angina from other types of chest pain? A. ECG B. exercise stress test C. coronary angiogram D. transesophageal echocardiogram

B

Pain has been described as whatever the person experiencing the pain says it is, existing whenever the patient says it does. This definition is problematic for the nurse when caring for what type of patient? A. a patient placed on a ventilator B. a patient with a history of opioid addiction C. a patient with decreased cognitive function D. a patient with pain resulting from severe trauma

C

in providing care for a patient hospitalized with an acute exacerbation of polycythemia vera, the nurse gives priority to which activity A. maintaining protective isolation B. promoting leg exercises adn ambulation C. protecting the patient from injury or falls D. promoting hydration with a large oral fluid intake

B

the male patient is admitted with a diagnosis of BPH. What urination characteristics would the nurse expect to assess in this patient? A. oliguria B. hesitancy C. hematuria D. proteinuria

B

Which manifestation is an indication that a patient is having a hypertensive emergency? A. symptoms of a stroke with an elevated BP B. systolic BP >180 and diastolic BP > 110 C. a sudden rise in BP accompanied by neurologic impairment D. Severe elevation of BP that occurs over several days or weeks

C

a patient admitted to the hospital following a generalized tonic-clonic seizure asks the nurse what caused the seizure. What is the best answer? A. there are so many factors that it is hard to tell B. epilepsy is an inherited disorder C. in seizures, some type of trigger causes sudden, abnormal bursts of electrical brain activity D. scar tissue in the brain alters the chemical balance causing uncontrolled electrical discharges

C

in promoting health maintenance for prevention of strokes, the nurse understands that the highest risk for the most common type of stroke is present in which people? A. african americans B. women who smoke C. Individuals with hypertension and diabetes D. those with high dietary fat intake

C

the nurse teaches the patient taking antiseizure drugs that the method most commonly used to measure compliance/ monitor for toxicity is: A. a daily seizure log B. urine testing C. blood testing D. EEG monthly

C

What is one of the most significant factors in determing when to start ART in a patient with HIV infection? A. whether they have high levels of HIV antibodies B. confirmation that the patient has HIV C. patient's readiness to commit to a lifelong, uncomfortable drug regimen D. whether the patient has a support system to help with treatment regimen

C.

When a patient reports chest pain, why must unstable angina be identified and rapidly treated? A.pain may be severe/disabling B. ECG changes and dysrhythmias may occur during an attack C. Rupture of unstable plaque may cause complete thrombosis of the vessel lumen D. Spasm of a major coronary artery may cause total occlusion of the vessel with progression to MI

C.

while caring for a patient who is at the nadir of chemotherapy, the nurse establishes the highest priority for nursing actions related to A. diarrhea B. grieving C. risk for infection D. inadequate nutritional intake

C.

The patient complains of wetting when she sneezes. What kind of incontinence is this? A. stress. B. urge C. micturition D. nocturia

A

Which medication should the nurse anticipate being used first in the ED for relief of severe respiratory distress related to asthma? A. prednisone orally B. tiotropium inhaler C. fluticasone inhaler D. albuterol nebulizer

D

A nurse is caring for a client who has type 1 diabetes mellitus. The nurse finds the client lying in bed, sweating, tachycardic, and reporting lightheadedness and palpitations. Which of the following complications should the nurse suspect? Hypoglycemia Nephropathy Hyperglycemia Ketoacidosis

Hypoglycemia Common manifestations of hypoglycemia include sweating, tachycardia, tremors, palpitations, hunger, and lightheadedness. Common manifestations of nephropathy include hypertension, lethargy, drowsiness, headache, and dry skin and mucous membranes. Common manifestations of hyperglycemia include dry skin and mucous membranes, rapid respirations, and changes in consciousness. Common manifestations of ketoacidosis include tachycardia, but also dry mucous membranes, altered consciousness, seizures, and hypotension.

What type of bleeding will a patient with peptic ulcer disease with a slow upper GI source of bleeding have? A. melena B. occult blood C. coffee-ground emesis D. profuse bright-red

melena

A nurse is preparing a client for surgery. She should begin preoperative teaching by exploring... what the client knows about the surgery. the client's usual coping mechanisms. the client's current home environment. which family members will help with postoperative care.

what the client knows about the surgery. The first step in client instruction is to determine the client's learning needs. The nurse does this by determining what the client needs to know, in this case, about the perioperative experience. All of the other options will be done, but they are not the first step in initiating a preoperative discussion.

A nurse is teaching a client who has recurrent urinary tract infections (UTIs) about prevention measures. Which of the following client statements indicates the need for further teaching? "I will need to wipe my perineal area from back to front after urination." "I will need to empty my bladder regularly and completely." "I will need to evacuate my bowels regularly." "I need to drink an adequate amount of liquid each day."

"I will need to wipe my perineal area from back to front after urination." Wiping the perineal area from back to front increases the risk for urethral contamination and a resulting UTI. All other options are appropriate UTI prevention measures.

A nurse is educating a client who is prescribed metformin (Glucophage) for type II diabetes mellitus. Which of the following information should the nurse include? "Take the medication with your meal." "You may crush or chew the medication." "Drinking alcohol is allowed when taking the medication." "Monitor for hypoglycemia when taking the medication."

"Take the medication with your meal." Taking the metformin with a meal or shortly thereafter is correct information for the nurse to include. Crushing or chewing metformin is not correct information for the nurse to include, because extended release medication should not be crushed or chewed. Drinking alcohol is not correct information for the nurse to include, because alcohol should be avoided when taking metformin. Monitoring for hypoglycemia is not correct information for the nurse to include, because the client is not at risk when metformin is taken in the recommended therapeutic dosage.

A nurse in a clinic is caring for a client who is postmenopausal and asks about medication to prevent osteoporosis. The nurse anticipates the client will be given a prescription for which of the following medications by the nurse practitioner? Levothyroxine (Synthroid) Calcitonin (Miacalcin) Raloxifene hydrochloride (Evista) Alendronate sodium (Fosamax)

. Raloxifene hydrochloride (Evista) Raloxifene hydrochloride is prescribed for prevention of osteoporosis in postmenopausal women. Levothyroxine is prescribed to treat hypothyroidism. Calcitonin is prescribed to delay bone resorption.in women who already have osteoporosis. Alendronate sodium is prescribed to increase bone mass in woman who already have osteoporosis.

A nurse is reinforcing teaching with a client who has diabetes mellitus. Which of the following should the nurse include in the teaching? (Select all that apply.) Reduce cholesterol and saturated fat intake. Increase physical activity and daily exercise. Enroll in a smoking-cessation program. Sustain hyperglycemia to reduce deterioration of nerve cells. Maintain optimal blood pressure to prevent kidney damage.

1, 2, 3 & 5. Diabetes Mellitus is associated with cardiovascular disease (CVD). Most clients die from a myocardial infarction or heart failure. Clients who have diabetes often have additional risk factors for cardiovascular disease, such as obesity, hypertension, dyslipidemia, and a sedentary lifestyle. The American Diabetes Association recommends that low-density lipoprotein (LDL) remain below 100 mg/dL. Diets high in saturated fat raise total cholesterol and LDL levels, which increase the risk for coronary artery disease. High-density lipoproteins (HDL) contain cholesterol as their primary core lipid and account for 20-30% of all cholesterol in the blood. HDLs carry cholesterol from peripheral tissues back to the liver to promote cholesterol removal. High triglycerides are associated with obesity, a sedentary lifestyle, and high carbohydrate intake. If the client is overweight, weight loss is indicated. Weight loss and increased physical activity are recommended to improve the lipid profile and to decrease the risk for heart disease. Renal tissue perfusion will be enhanced with exercise as well. Cigarette smoking can cause vasoconstriction of vessels. Clients who have diabetes have increased risk for coronary artery disease. Vasoconstriction causes further narrowing of vessels and increases blood pressure. Vasoconstriction increases risk for myocardial infarction and cerebrovascular accident (CVA). Nephropathy is a pathologic change in the kidney that reduces kidney function and leads to kidney failure. Diabetes is the leading cause of end-stage kidney disease. Risk factors for nephropathy include a 10-15 year history of diabetes mellitus, poor glucose control, and uncontrolled hypertension. Both systolic and diastolic hypertension speed the progression of diabetic nephropathy. Glycosylated hemoglobin (HbA1c) is measured to assess long-term glycemic control. To reduce complications related to diabetes, the HbA1c should be less than 7%.

A nurse is creating a plan of care for a client who has a tonic-clonic seizures disorder. Which of the following seizure precautions should the nurse implement? (Select all that apply.) Provide a suction setup at the bedside. Elevate the side rails when in bed. Place the bed in the lowest position Keep an oxygen setup at the bedside. Furnish restraints at the bedside.

1, 2, 3 & 4. Provide a suction setup at the bedside, Elevate the side rails when in bed, Place the bed in the lowest position and Keep an oxygen setup at the bedside. The nurse should provide a suction setup at the bedside to do oral suctioning following the seizure to prevent aspiration. The nurse should also implement elevation the rails of the bed to prevent a fall during a seizure. The bed should be placed in the lowest position to prevent injury if a fall should occur during a seizure. And the nurse should keep an oxygen setup at the bedside to administer oxygen during a seizure. Furnish restraints at the bedside is incorrect. The client should not be restrained during a seizure.

A nurse is preparing a presentation at a community center about knee disorders and injuries. The nurse should include which of the following as risk factors for developing osteoarthritis? (Select all that apply.) Obesity Family history of osteoarthritis Calcium deficiency Aging Regular, strenuous exercise

1, 2, 4 & 5. Obesity, Family history of osteoarthritis, aging and Regular, strenuous exercise Obesity is a risk factor for osteoarthritis, as it increases the load of the body's weight over time. There is a genetic predisposition for developing osteoarthritis. Too little calcium leads to osteoporosis, not osteoarthritis. Aging is a risk factor for osteoarthritis, as the joints bear the load of the body's weight over time. Strenuous exercise and repetitive motion can result in osteoarthritis.

A charge nurse is assessing a newly licensed nurse's understanding of the need to administer 0.9% sodium chloride with packed RBC for a client who has anemia. Which of the following statements by the newly licensed nurse indicates an understanding of this intervention? "These products should be administered together to decrease the risk of hemolysis." "0.9% sodium chloride decreases the risk of an allergic reaction during the transfusion." "When these products are administered together the risk for circulatory overload is decreased." "0.9% sodium chloride decreases the risk of bacterial contamination during a transfusion."

1."These products should be administered together to decrease the risk of hemolysis." The nurse should administer 0.9% sodium chloride with blood products to decrease the risk for clotting and hemolysis. Although 0.9% sodium chloride is the fluid of choice to administer with blood products, it does not decrease the risk for an allergic reaction or circulatory overload and it does not decrease the risk of bacterial contamination.

A nurse is performing teaching with a client who has newly diagnosed type 2 diabetes mellitus. The nurse should recognize that the client understands the teaching when he identifies which of the following manifestations of hypoglycemia? (Select all that apply.) Polyuria Vertigo Polydipsia Tachycardia Acetone breath Moist, clammy skin

2, 4 & 6. Vertigo, Tachycardia & Moist, clammy skin A client who is newly diagnosed with type 2 diabetes mellitus should be taught to recognize the manifestations of hypoglycemia (decreased blood sugar) that may occur as a result of an insulin reaction, inadequate intake of glucose, or increased exercise. Manifestations of hypoglycemia include vertigo (dizziness), tachycardia and moist, clammy skin. Manifestations of hyperglycemia include polyuria (excessive urination) and polydipsia (excessive thirst).

A nurse working in a dermatologist's office is planning an educational session regarding skin cancer. When discussing risk factors the nurse should include which of the following? (Select all that apply.) Being dark-skinned Age under 40 years. Overexposure to ultraviolet light Chronic skin irritations Genetic predisposition

3, 4 & 5. Overexposure to ultraviolet light, Chronic skin irritations and Genetic predisposition Light-skinned individuals, individuals over 60 years, overexposure to ultraviolet light, chronic skin irritations and genetic predisposition are risk factors for developing skin cancer.

A nurse is caring for a client who is requesting prescription pain medication. Which of the following actions should the nurse perform first? Reposition the client. Administer the medication. Determine the location of the pain. Review the effects of the pain medication.

3. Determine the location of the pain. Using the nursing process, assessment of the location of the pain is the priority action by the nurse.

A nurse at an ophthalmology clinic is caring for a client who has open-angle glaucoma. The client is started on a treatment regimen of timolol (Timoptic) and pilocarpine (Pilocar) eye drops. The nurse should understand that these medications will be administered... 1. when the client is experiencing eye pain. 2. until the client's intraocular pressure returns to normal. 3. on a regular schedule for the rest of the client's life. 4. for approximately 10 days, followed by a gradual tapering off.

3. on a regular schedule for the rest of the client's life. Medications prescribed for glaucoma are intended to enhance aqueous outflow, or decrease its production, or both. The client must continue the eye drops on an uninterrupted basis for life. All other options can worsen the client's condition.

A nurse is caring for a client who has congestive heart failure (CHF) and was started on digoxin (Lanoxin). Which of the following client statements should alert the nurse that the client may be experiencing a side effect of digoxin? "I can walk a mile a day." "I've had a backache ever since I got home." "I've lost half a pound in the last 2 days." "I feel nauseated and have no appetite."

4. "I feel nauseated and have no appetite." Anorexia, nausea, vomiting, and abdominal discomfort are early signs of digitalis toxicity. Improving the client's cardiac output, which in turn will improve the client's exercise tolerance, is a desired response to digoxin. Backaches are not related to the use of digoxin for CHF. A weight loss of half a pound is not significant and is not likely related to the use of digoxin.

A nurse is caring for a client who is diagnosed with a cerebrovascular accident (CVA, stroke). Which of the following actions should be implemented to prevent deep-vein thrombosis (DVT)? Massage lower extremities daily. Check for positive Homans' sign. Monitor the client's level of consciousness. Place sequential compression devices bilaterally.

4. Place sequential compression devices bilaterally. DVT is the most common type of thrombophlebitis. DVT usually is treated using a combination of rest, anticoagulant therapy, and sequential compression devices (SCDs). The use of SCDs helps prevent blood stasis by promoting circulation. The SCDs should be measured from the middle of the foot to just below the knee or thigh and are worn while in bed. The nurse should refrain from massaging lower extremities in the event that the client has an existing DVT. Massage can promote the dislodging of the Pain in the calf upon dorsiflexion of the foot (positive Homans' sign) appears in only a small percentage of clients who have DVT, and false positive findings are common. Therefore, checking for Homans' sign is not advised.

A 72 year old woman is diagnosed with diabetes. What does the nurse recognize about the management of diabetes in the older adult? A. it is more difficult to achieve strict glucose control than in younger patients B. treatment is not warranted unless the patient develops severe hyperglycemia C. It does not include treatment with insulin because of limited dexterity and vision. D. It usually requires that a younger family member be responsible for care of the patient

A

Which classification of chemo drugs is cell cycle phase-nonspecific, breaks the DNA helix that interferes with DNA replication, and crosses the blood-brain barrier? A. nitrosureas B. antimetabolites C. mitotic inhibitors D. antitumor antibiotics

A

While receiving a unit of packed RBCs, the patient develops chills and a temperature of 102.2. What is the priority nursing action? a. stop the transfusion and instill normal saline b. notify the health care provider and the blood blank c. add a leukocyte reduction filter to the blood administration set d. recognize this as a mild allergic transfusion reaction and slow the transfusion

A

A nurse is caring for a client after a craniotomy for pituitary tumor who has developed diabetes insipidus. The client is receiving vasopressin (Pitressin). The desired response to the medication is evident when the nurse observes which of the following findings? A decrease in blood sugar. A decrease in blood pressure. A decrease in urine output. A decrease in specific gravity.

A decrease in urine output. The major manifestations of diabetes insipidus are excessive urination and extreme thirst. Pitressin is used to control frequent urination, increased thirst, and loss of water associated with diabetes insipidus. A decreased urine output is the desired response.

A patient with advanced colorectal cancer has continuous, poorly localized abdominal pain at an intensity of 5 on a scale of 0 to 10. How does the nurse teach the patient to use pain medications? A. on an around-the clock schedule B. As often as necessary to keep pain controlled C. by alternating two different types of drugs to prevent tolerance D. when the pain cannot be controlled by distraction/relaxation

A.

A patient with diabetes is learning to mix regular insulin and NPH insulin in the same syringe. The nurse determines that additional teaching is necessary when the patient does what? A. withdraws the NPH dose into the syringe first B. injects air equal to the NPH dose into the NPH vial first C. removes any air bubbles after withdrawing the first insulin D. adds air equal to the insulin dose into the regular vial and withdraws the dose

A.

A patient with trigeminal neuralgia has moderate to severe burning and shooting pain. In helping the patient manage the pain, the nurse recognizes what about this kind of pain? A. includes treatment with adjuvant analgesics B. Will be chronic and require long term treatment C. Responds to small to moderate around the clock doses of oral opioids D. Can be well controlled with NSAIDs

A.

After which diagnostic study should the nurse observe the patient for symptoms of a pneumothorax? A. Throacentesis B. Pulmonary function test C. Ventilation-perfusion scan D. PET scan

A.

During care of the patient with SIADH, what should the nurse do? A. monitor neurologic status at least every 2 hours B. Teach the patient receiving treatment with diuretics to restrict sodium intake C. Keep the head of the bed elevated to prevent ADH release D. Notify HCp if the patient's BP decreases more than 20 mmHG from baseline

A.

During initial chemotherapy, a patient with leukimia develops hyperkalemia and hyperuricema. The nurse recognizes this symptoms as an oncologic emergency and anticipates that priority treatment will be to A. increase urine output with hydration therapy B. establish ECG monitoring C. administer a biphosphonate such as panidronate D. restrict fluids and administer a hypertonic solution such as sodium chloride

A.

A patient comes to the clinic and requests testing for HIV infection. Before adminsitering testing, what is the most important for the nurse to do? A. Ask the patient to identify all sexual partners. B. Determine when the person thinks exposure occurred C. explain that all test results must be repeated at least twice to be valid D. discuss prevention practices to prevent transmission of HIV to others

B

A patient has a paltelet count of 50,000 and is diagnosed with ITP. What does the nurse anticipate that initial treatment will include? A. splenectomy B. corticosteroids C. administration of platelets D. immunosuppressive therapy

B

A patient with Addison's disease comes to the ED with complaints of N/V, diarrhea, fever. What interprofessional care should the nurse expect? A. IV administration of vasopressors B. IV administration of hydrocortisone C. IV administration of dextrose 5W and 20 KCL D. parenteral injections of ACTH

B

A patient with a hematologic disorder has a smooth, shiny tongue. Which laboratory result would the nurse expect to see? A. Neutrophils 45% B. Hgb 9.6 C. WBC count 13,500 D. RBC 6.4 million

B

A patient with a tremor is being evaluated for Parkinson's. The nurse explains to the patient that Parkinson's can be confirmed by A. CT and MRI scans B. relief of symptoms with administration of dopaminergic agents C. presence of tumors that increase during voluntary movement D. cerebral angiogram that reveals atherosclerosis

B

A patient with heart failure has atrial fibrillation and a left ventricular ejection fraction of 18%. To decrease the risk of complications from these conditions, what drug does the nurse anticipate giving? A. diuretics B. anticoagulant C. B-adrenergic blocker D. potassium supplement

B

A patient with hypothyroidism is treated with Synthroid. When teaching the patient about the therapy, the nurse a. explains that caloric intake must be reduced when drug therapy is started b. provides written instruction for all information related to the medication therapy c. assures the patient that a return to normal function will occur with replacement therapy d. informs the patient that medications must be taken until hormone balance is reestablished

B

A patient with type 1 diabetes uses 20 U of Novolon (NPH/Regular) in the morning and at 6 pm. When teaching the patient about this regimen, what should the nurse emphasize? A. hypoglycemia is most likely to occur before the noon meal B. a set meal pattern with a bedtime snack is necessary to prevent hypoglycemia. C. flexibility in food intake is possible because insulin is available 25 hours a day D. premeal glucose checks are required to to determine needed changes in daily dosing

B

The patient is diagnosed with a superficial vein thrombosis (SVT). Which characteristic should te nurse know about SVT? A. embolization to lungs results in death B. clot may extend to deeper veins if untreated C. vein is tender to pressure and there is edema D. typically found in the iliac, inferior, or superior vena cava

B

What nursing action is the most important for the patient with age-related dry macular degeneration (AMD)? A. teach the patient how to use topical eyedrops for treatment B. emphasize the use of vision enhancement techniques to improve what vision is present C. encourage the patient to undergo laser treatment to slow the deposit of extracellular debris d. explain that nothing can be done for the patient's vision because there is no treatment.

B

What should the goals of nutrition therapy for the patient with type 2 diabetes include? A. ideal body weight B. normal serum and lipid levels C. a special diabetic diet using dietic foods D. 5 small meals a day and a bedtime snack

B

When caring for a patient with primary hyperaldosteronism, the nurse would question HCP's prescription for which drug? A. Ketoconazole B. Furosemide (lasix) C. Eplerenone (inspra) D. Spironolactone

B

a patient is admitted to the emergency department with an acute asthma attack. which patient assessment if of greatest concern to the nurse? A. the presence of a pulsus paradoxus B. markedly diminished breath sounds with no wheezing C. a respiratory rate of 34 and increased pulse and BP D. use of accessory muscles of respiration and a feeling of suffocoation

B

a patient with chronic heart failure is treated with hydrochlorothiazide, digoxin, and lisinopril. To prevent the risk of digitalis toxicity with these drugs, what is te most important that the nurse monitor for this patient? A. heart rate B. Potassium levels C. BP D. Gi function

B

during physical assessment of a patient with thrombocytopenia, the nurse should expect to find A. sternal tenderness B. petechiae and purpura C. jaundiced sclera and skin D. tender, enlarged lymph nodes

B

during the assessment, the nurse identifies crackles in the lungs and an S3 heart sound. Which complication of MI should the nurse suspect and further investigate? A.pericarditis B. heart failure C. ventricular aneuyrsm D. papillary muscle dysfuncrion

B

following a mastectomy, a patient develops lymphedema on the affected arm. What does the nurse teach the patient to do? A. avoid skin-softening agents on their arm B. protect the arm from any type of trauma C. abduct and adduct the arm at the shoulder hourly D. Keep the arm positioned in a straight and dependent alingment

B

The nurse explains to a client that the main reason a back rub is used as therapy to relieve pain is because the massage: Blocks pain impulses from the spinal cord to the brain. Blocks pain impulses from the brain to the spinal cord. Stimulates the release of endorphins. Distracts the client's focus on the source of the pain

Blocks pain impulses from the spinal cord to the brain. A back rub stimulates the large-diameter cutaneous fibers, which block transmission of pain impulses from the spinal cord to the brain. It does not block the transmission of pain impulses or stimulate the release of endorphins. A back rub may distract the client, but the physiologic process of fiber stimulation is the main reason a back rub is used as a therapy for pain relief.

A carotid endarterectomy is being considered as a treatment for a patient who has had several TIAs. The nurse explains to the patient that this surgery a. is used to restore blood to the brain following an obstruction of a cerebral artery b. involves intracranial surgery to join a superficial extracranial artery to an intracranial artery c. involves removing an atherosclerotic plaque in the carotid artery to prevent an impending stroke d. is sued to open a stenosis in a carotid artery with a balloon and stent to restore cerebral circulation

C

A patient taking insulin has recorded fasting glucose levels above 200 on awakening for the last five mornings. What should the nurse advise the patient to do first? A.icnrease the evening dose of insulin to prevent the dawn phenomenon B. use a single-dose insulin regimen with an intermediate-acting insulin C. monitor the glucose levels at bedtime, between 2 Am and 4 Am and on arising D. decrease the insulin dosage to prevent night hypoglycemia and the Somogyi effect

C

An antidepressant is prescribed for a patient with chronic pain from fibromyalgia. The patient states that she is in pain, not depressed. What is the nurse's best response to the patient? A. Antidepressants will improve the patient's attitude and prevent a negative emotional response to the pain. B. Chronic pain almost always leads to depression and this is a preventative measure. C. Some antidepressants relieve pain by releasing neurotransmitters that prevent pain impulses from reaching the brain. D. Certain antidepressants are metabolized in the liver to substances that numb the ends of nerve fibers preventing the onset of pain

C

Care of the patient with an NSTEMI is different than that of a patient with a STEMI in that NSTEMI is more frequently initially treated with what? A. PCI B. CABG C. acute intensive drug therapy D. reperfusion therapy with thrombolytics

C

Corticosteroids are associated with the development of peptic ulcers because of which probably pathophysiologic mechanism? A. enzyme urease is produced B. secretion of HCL is increased C. rate of mucous cell renewal is decreased D. synthesis of mucus and prostaglandins is inhibited

C

During assessment of a patient admitted to the hospital with an acute exacerbation of MS, what should the nurse expect to find? A. tremors and dysphasgia B. bowel/bladder incontinence and loss of memory C. motor impairment, visual disturbances, and paresthesia D. excessive involuntary movements, hearing loss, and ataxia

C

The "rapid" HIV antibody test is performed on patients at high risk for HIV infection. What should the nurse explain about this test? A. it measures the activity of HIV and reports viral load as real numbers B. is highly reliable, and in 5 minutes the patient will know if they have HIV C. if the results are positive, another blood test and a return appointment for results are necessary D. it detects drug-resistant viral mutations that are present in viral genes to evaluate resistance to antiretroviral drugs

C

The hospital is very busy today. Which care could the nursing RN delegate to the UAP for a patient with VTE? A. assess the patient's use of herbs B. measure the patient for elastic compression stockings C. remind the patient to flex and extend the legs and feet every 2 hours D. teach the patient to call the ERS with signs of pulmonary embolus

C

The patient has a lack of comprehension of both verbal and written language. Which type of communication difficulty does this patient have? A. dysarthria B. fluent dysphasia C. receptive aphasia D. expressive aphasia

C

Upon admission of a patient to the PACU, the nurse's priority assessment is A. vital signs B. surgical site C. respiratory adequacy D. level of consciousness

C.`

To promote effective coughing, deep breathing, and ambulation in the postoperative patient, it is most important for the nurse to A.teach controlled breathing B. explain the rationale for these activities C. provide adequate and regular pain meds D. use incentive spirometer as motivation

C>

A nurse is caring for a client who was admitted to the hospital with congestive heart failure (CHF) and is taking digoxin (Lanoxin) 0.25 mg daily. The client refused breakfast and is complaining of nausea and generalized weakness. Which of the following actions should the nurse perform first? Check the client's vital signs. Request a dietitian consult. Suggest that the client rests before eating the meal. Request an order for an antiemetic.

Check the client's vital signs. It is possible that the client's nausea is secondary to digoxin toxicity. By obtaining vital signs, the nurse can assess for bradycardia, which is a symptom of digoxin toxicity. The nurse should withhold the drug and call the provider if the client has bradycardia. All other options may be appropriate, but are not the priority action.

A nurse is caring for a client who is being treated with cisplatin (Platinol) for cancer. After several treatments, the client begins to report fatigue. Which of the following nursing actions is appropriate? Check the results of the client's most recent complete blood cell count. Speak with the client's spouse about the dangers of prolonged immobility. Evaluate the client for further signs of depression. Suggest that the client set up daily exercise plans for the morning

Check the results of the client's most recent complete blood cell count. The client is likely to be anemic as a result of myelosuppression (bone marrow suppression) from the chemotherapy. The client may require treatment for the anemia (transfusion, medication) and further chemotherapy may need to be delayed until the blood counts are higher. The other options do not directly address the most likely physical cause of the client's symptoms.

In addition to smoking cessation, what treatment is included for COPD to slow the progression of the disease? A. Use of bronchodilator drugs B. Use of inhaled corticosteroids C. Lung volume reduction surgery D. Prevention of respiratory tract infections

D

The patient does not want to experience the risks of insulin. The HCP told him a medication will be prescribed that will increase insulin synthesis, and release from the pancreas, inhibit glucagon secretion, and slow gastric emptying. The nurse knows this is which medication that will have to be injected? A. dopamine receptor agonist, bromocriptine (cycloset) B. DPP-4 inhibitor, sitagliptin C. SGLT2 inhibitor, canagliflozin D. glucagon-like peptide receptor agonsit, exenatide extended release

D

To help prevent embolization of the thrombus in a patient with VTE and severe edema/limb pain, the nurse teaches the patient to do what FIRST A. dangle feet over the edge of the bed q2-3hr B. ambulate around the bed 3-4x a day C. keep the affected leg elevated above the level of the heart D. maintain bed rest until edema is relieved and anticoagulation is established

D

What is the rationale for treating acute exacerbations of peptic ulcer disease with NG intubation? A. stop spillage of GI contents into the peritoneal cavity B. remove excess fluids and undigested food from the stomach C. feed the patient the nutrients missing from the lack of ingestion D. remove stimulation for HCL acid and pepsin secretion by keeping the stomach empty

D

When replacement therapy is started for a patient with long-standing hypothyroidism, it is most important for the nurse to monitor the patient for a. insomnia b. weight loss c. nervousness d. dysrhythmias

D

At the end of the surgery, the perioperative nurse evalautes the patient's response to the nursing care delivered during the perioperative period. What reflects a positive outcome related to the patient's physical status? A. patient's right to privacy is maintained throughout the procedure B. patient's care is consistent with the preplanned perioperative plan of care C. patient receives consistent and comparable care regardless of the surgical setting D. patient's respiratory function is consistent with or improved from baseline levels established preoperatively

D.

Goals for patient safety in the OR include the Universal Protocol. What is included in this protocol? A. all surgical centers of any type must submit reports on patient safety infractions to accrediation agencies. B. members of the surgical team stop whatever they are doing to check that all sterile items have been prepared properly C. Members of the surgical team pause right before surgery to meditate for 1 minute to decrease stress and possible errors D. a surgical timeout is performed just before the procedure is started to verify patient identify, surgical procedure, and surgical site

D.

Prophylactic measures that are routinely used as early as possible in HIV infection to prevent opportunistic and debilitating secondary problems include administration of: A. isoniazid to prevent tuberculosis B. zoster virus vaccination to prevent shingles C. trimethoprim/sulfamethazole for toxoplasmosis D. vaccines for pneumococcal pneumonia, influenza, and hepatitis a and b

D.

The nurse explains to the patient newly diagnosed with MS that the diagnosis is made primarily by A. spinal x-ray findings B. T cell analysis of the blood C. analysis of CSF D. history and clincial manifestations

D.

A nurse is providing education on site rotation for insulin injections to a client who is newly diagnosed with type I diabetes mellitus. Which of the following reasons should the nurse include in the teaching for site rotation? Decreases the risk for lipoatrophy with insulin injections. Includes administering injections within 1 inch of the umbilicus. Reduces the risk for infection associated with insulin injections. Minimizes the pain associated with insulin injections.

Decreases the risk for lipoatrophy with insulin injections. The nurse should educate the client to rotate injection sites in the same anatomic area to decrease lipoatrophy, which is a loss of fat under the skin in the area of the injections. The nurse should educate the client to administer the insulin injections at least 2 inches from the umbilicus. The nurse should educate the client that site rotation does not reduce infection associated with insulin injection. The nurse should educate the client that site rotation is not to minimize pain associated with insulin injections.

A client who is scheduled for lithotripsy asks the nurse about conditions that can contribute to the formation of renal calculi. The nurse explains that risk factors include which of the following: Protein in the urine Dehydration Iron deficiency Obesity

Dehydration Inadequate fluid intake and urinary stasis can promote the formation of calculi. The other options do not place the client at risk for renal calculi.

A nurse is preparing teaching for a female client who smokes, is obese, and has hypertension. In establishing health promotion goals for the client, the nurse should recognize that which of the following is an inappropriate recommendation for the client? Eliminate sodium from the diet. Exercise moderately three times a week. Start a weight reduction diet. Use nicotine patches to stop smoking.

Eliminate sodium from the diet. Eliminating sodium from the client's diet is not a reasonable goal. A diet appropriate for a hypertensive client will restrict intake of sodium, not eliminate it. Exercising moderately three times a week is an appropriate initial goal for an obese, hypertensive client. Starting a weight reduction diet and using nicotine patches to stop smoking are realistic goals for this client.

A nurse is planning care for a client following surgery who is having headaches due to receiving spinal anesthetic. Which of the following is included in the plan of care? Encourage increased intake of fluids. Encourage increased physical activity. Maintain the client in high Fowler's position. Apply an ice bag at the injection site of the spinal anesthetic.

Encourage increased intake of fluids. Increased oral fluid intake promotes increases intracranial pressure which may relieve spinal headaches. Decreased, not increased, physical activity is helpful in relieving spinal headaches. Keeping the client flat, not in high fowlers, is helpful in relieving spinal headaches. A cold pack at the injection site may decrease pain in this area but does not relieve a spinal headache.

A nurse is caring for a client who has chronic obstructive pulmonary disease (COPD). The client tells the nurse, "I can feel the congestion in my lungs, and I certainly cough a lot, but I can't seem to bring anything up." Which of the following actions should the nurse take to help this client with tenacious bronchial secretions? Maintaining a semi-Fowler's position as much as possible Administering oxygen via nasal cannula at 2 L per min Helping the client select a low-salt diet Encouraging the client to drink eight glasses of water daily

Encouraging the client to drink eight glasses of water daily COPD is a term that is used for two closely-related diseases of the respiratory system: chronic bronchitis and emphysema. Maintaining hydration through the consumption of adequate fluids will help to liquefy the tenacious (thick) secretions. It is advised that the client drink six to eight glasses of fluid (preferably water) daily to keep the bronchial secretions thin.

A nurse is caring for a client who has heart failure and has a potassium level of 2.4 mg/dL. An adverse effect of which of the following medications is a possible cause of this potassium level? Furosemide (Lasix) Nitroglycerin (Nitro-Bid) Metoprolol (Lopressor) Spironolactone (Aldactone)

Furosemide (Lasix) Anorexia, nausea, vomiting, and abdominal discomfort are early signs of digitalis toxicity. Improving the client's cardiac output, which in turn will improve the client's exercise tolerance, is a desired response to digoxin. Backaches are not related to the use of digoxin for CHF. A weight loss of half a pound is not significant and is not likely related to the use of digoxin.

A nurse is teaching a client with an estrogen-receptor (ER)-positive breast cancer tumor who has a new prescription for tamoxifen citrate (Nolvadex). Which of the following should be included in the teaching regarding adverse effects of this medication? Hot flashes are common. Weight loss is noted. An increase in appetite may occur. Constipation is common.

Hot flashes are common. Hot flashes are a common adverse effect. Weight gain, not weight loss is noted. Loss of appetite, not an increase, may occur. And constipation is not an adverse effect of this medication.

A nurse is caring for a client who has COPD. When developing this client's plan of care, the nurse should include which of the following interventions? Restrict the client's fluid intake to less than 2 L/day. Encourage the client to use the upper chest for respiration. Have the client use the early-morning hours for exercise and activity. Instruct the client to use pursed-lip breathing.

Instruct the client to use pursed-lip breathing. Pursed-lip breathing lengthens the expiratory phase of respiration and also increases the pressure in the airway during exhalation. This reduces airway resistance and decreases trapped air for clients who have COPD. Unless the client has another medical disorder that warrants fluid restriction, he should drink 2 to 3 L of fluid/day. Clients who have COPD should breathe from the diaphragm, not the upper chest. Clients who have COPD have poor exercise tolerance in the early morning due to the pulmonary secretions that accumulate when the person is recumbent.

A nurse is caring for a client with peripheral arterial disease (PAD). Which of the following symptoms is typically the initial reason clients with PAD seek medical attention? Intermittent claudication Dependent rubor Rest pain Foot ulcers

Intermittent claudication Intermittent claudication is ischemic pain that is precipitated by exercise, resolves with rest, and is reproducible. The pain associated with claudication arises when cellular oxygen demand exceeds supply, and is typically the initial reason clients with PAD seek medical attention. Dependent rubor, rest pain and foot ulcers are manifestations that occur in later stages of chronic PAD.

A home health nurse is caring for an older adult client who just had cataract surgery. The nurse should include which of the following client instructions in the care plan? Rest in bed for at least 2 days. Keep your head up and straight. Deep-breathe and cough four times a day. Lie on the side of the surgery when in bed.

Keep your head up and straight. Keeping the head straight and avoiding looking down prevent increasing intraocular pressure. Remaining in bed after cataract surgery puts the client at risk for the hazards of immobility. The client should resume previous mobility as soon as 1 hr after surgery. Deep breathing is helpful for any client, but coughing increases intraocular pressure. Thus, the client should avoid coughing after cataract surgery. Most clients have some mild discomfort in the eye after cataract surgery, so it is generally more comfortable for clients to lie on the other side or on their back when resting.

A nurse is caring for a client who has prostate cancer. The nurse should expect the provider to prescribe which of the following medications for this client? Leuprolide (Lupron) Cyclophosphamide (Cytoxan) Finasteride (Proscar) Tamoxifen (Nolvadex_

Leuprolide (Lupron) Leuprolide treats cancer of the prostate hormonally. It antagonizes the androgens that androgen-dependent neoplasms require. Cyclophosphamide treats leukemia, multiple myeloma, lymphomas, and head, ovary, breast, and lung cancer. Finasteride treats benign prostatic hypertrophy and also helps reduce the risk of prostate cancer. Tamoxifen treats breast cancer.

A nurse is caring for a client who receives furosemide (Lasix) to treat heart failure. Which of the following laboratory values should the nurse be sure to monitor specifically for this client? Potassium Albumin Chloride Bicarbonate

Potassium Furosemide is a loop diuretic that promotes the excretion of potassium. The nurse should monitor the client's potassium level to watch for hypokalemia.

A nurse is caring for a client who has returned from the surgical suite following surgery for a fractured mandible. The client has had intermaxillary fixation to repair and stabilize the fracture. The nurse should recognize that the most important goal in the immediate postoperative period is to do which of the following? Prevent aspiration. Ensure adequate nutrition. Promote oral hygiene. Relieve the client's pain.

Prevent aspiration. Because the jaws are wired together, if the client should vomit, aspiration of the emesis is a possibility. Therefore, the client should be given medication for nausea, as indicated, and wire cutters should be kept at the bedside, in case of vomiting. The client will be NPO (nothing by mouth) initially after surgery until the gag reflex has returned. Once the client is able to eat, the client will advance to a calorie-appropriate, high-protein liquid diet. While it is important that the client receive frequent mouth cleaning and adequate nutrition and may be in pain and will need to be medicated, these are not the most important concerns in the immediate postoperative period.

A nurse is caring for a client admitted with a diagnosis of hyperthyroidism. The client reports a weight loss of 5.4 kg (12 lb) in the last 2 months despite increased appetite. Additional symptoms reported include increased perspiration, fatigue, menstrual irregularity, and restlessness. Which of the following measures should the nurse include in the client's plan of care to prevent a thyroid crisis? Provide a quiet, low-stimulus environment. Administer aspirin as prescribed for any sign of hyperthermia. Maintain the client's NPO status. Observe the client carefully for signs of hypocalcemia.

Provide a quiet, low-stimulus environment. Thyroid crisis can occur in response to a stressor, so the client should not be exposed to other clients who have active infections or an environment that is noisy and stimulating. In clients who have hyperthyroidism, aspirin displaces the thyroid hormone from plasma proteins and results in active thyroid hormone in the blood, which may exacerbate a thyrotoxic crisis. The client should be encouraged to eat a high-protein, high-calorie diet to maintain weight and prevent negative nitrogen balance. Hypocalcemia is a clinical finding in hypoparathyroidism, and calcium levels do not play a role in preventing thyrotoxic crisis.

The nurse is conducting a class on Parkinson's disease for a group of family members. Which of the following information should the nurse include in the teaching? Provide client supervision. Limit client physical activity. Speak loudly to the client. Leave the television on continuously.

Provide client supervision. The nurse's information should include providing client supervision to create a safe and respectful environment. It should also include providing exercise program to improve mobility, alternated with periods of rest, not limiting activity, speaking clearly and in a normal tone to the client, not loudly and should include decreasing excess environmental noise to increase the client's ability to concentrate on listening so, the television should be off.

A client returns from the postanesthesia care unit after a subtotal thyroidectomy. The client's vital signs are stable and her dressing is dry. The nurse should assist the client in maintaining which of the following positions? High Fowler's with her neck extended High Fowler's with her neck in a neutral position. Semi-Fowler's with her neck extended Semi-Fowler's with her neck in a neutral position

Semi-Fowler's with her neck in a neutral position Semi-Fowler's is the most comfortable position for a client who has had thyroid surgery. Neck flexion could compromise the airway, and neck extension could place excessive tension on the operative area and the sutures. A neutral position is essential

A nurse is planning care for a client who has pernicious anemia. Which of the following should the nurse implement? Vitamin B12 (cyanocobalamin) injections. Iron supplements. Blood transfusions. Vitamin B6 (pyridoxine) supplements.

Vitamin B12 (cyanocobalamin) injections. The nurse should administer Vitamin B12 injections to treat pernicious anemia when diet fails to improve the anemia due to poor absorption. Iron supplements treat iron deficiency anemia.

during the physical assessment of the patient with severe anemia, which finding is of the most concern to the nurse? A. anorexia B. bone pain C. hepatomegaly D. dyspnea at rest

D

a 38 year old woman is newly diagnosed with MS and asks the nurse what is going to happen to her. What is the best response by the nurse? A. you will have either periods of attacks and remissions or progression of nerve damage over time B. you will need to plan for a continuous loss of movement, sensory functions, and mental capabilities C. you will most likely have a steady course of chronic progressive nerve damage that will change your personality D. it is common for people with MS to have an acute attack of weakness and then to not have symptoms for years

A

A 60 year old patient is being prepared for outpatient cataract surgery. When obtaining admission data, what should the nurse expect to find in the patient's history? A. a painless, sudden, severe loss of vision B. blurred vision, colored halos around lights, and eye pain C. a gradual loss of vision with abnormal color perception and gaze D. light flashes and a cobweb in the field of vision

C

A patient breast tumor originates from embryonal ectoderm. It has moderate dysplasia and moderately differentiated cells. It is a small tumor with minimal lymph node involvement and no metastases. What is the best description of this tumor? A. sarcoma, grade II, T3/N4/M0 B. leukimia, grade I, T1/N2/M1 C. carcinoma, grade II, T1/N1/M0 D. lymphoma, grade III, T1/N0/M0

C

A patient with DI is treated with nasal desmopression. The nurse recognize that the drug is not having an adequate therapeutic effect the the patient experiences a. headache and weight gain b. nasal irritation and nausea c. a urine specific gravity of 1.002 d. an oral intake greater than urinary output

C

A patient with mild iatrogenic Cushing's syndrome is on an alternate-day regimen of corticosteroid therapy. What does the nurse explain to the patient about this regimen? A. it maintains normal adrenal hormone balance B. It prevents ACTH release from the pituitary gland C. It minimizes hypothalamic-pituitary adrenal suppression D. It provides a more therapeutic effect

C

A patient who recently had a calcium oxalate renal stone had a bone density study, which showed a decrease in her bone density. What endocrine problem could this patient have? A. SIADH B. hypothyroidism C. cushing's D. hyperparathyroidism

D

Why do opportunistic infections develop in an individual with AIDs? A. side effects of drug treatment of AIDs B. sexually transmitted to individuals during exposure to HIV C. characteristic individuals with stimulated B and T lymphocytes D. these infections or tumors occur in a person with an incompetent immune system

D.

when obtaining a health history from a 72 year old man with PAD of the lower extremities, the nurse asks about a history of related conditions, including A. venous thrombosis B. venous stasis ulcers C. pulmonary embolism D. CAD

D.

which indirect thrombin inhibitor is only administered subq and does not need routine coagulation tests? A. warfarin B. unfractioanted heparin C. Hirudin derivatices D. low molecular weight heparin (Lovenox/enoxaparin)

D.

A nurse is providing teaching for a client who is preparing for a below the knee amputation. Which of the following statements is appropriate regarding the postoperative placement of a prosthesis? "The prosthesis will be in place immediately following surgery to... improve your ability to ambulate sooner." decrease the chance of phantom limb pain." decrease the frequency of dressing changes." improve the fit of the prosthesis."

improve your ability to ambulate sooner." The nurse correctly explains that the purpose of a prosthesis immediately following surgery is to promote postoperative ambulation.


संबंधित स्टडी सेट्स

accounting 201 ch 6: inventory and cost of goods sold

View Set

Elimination PREP U (NUR 2 TEST 2)

View Set

Ch 3 The Human Body: A Nutrition Perspective

View Set

Ch 18 Cost Behavior and Cost Volume Profit Analysis

View Set

Network+ Guide to Networks 7th ed. Quiz Ch. 12

View Set

NUR 304 Chapter 49 Concepts of Care for Patients with Inflammatory Intestinal Conditions

View Set

Business Management and Administration Career Cluster

View Set

American History Unit 2: Lesson 4 - The Plains Indian Wars

View Set